My Praxis II Elem Edu Practice test Questions

Ace your homework & exams now with Quizwiz!

The standard enthalpy of formation of hydrogen peroxide is -187.8 kcal/mole ( for the reaction H2(g) + O2(g) → H2O2(l) ). The standard enthalpy of formation of water is -285.8 kcal/mole

( for the reaction H2(g) + ½ O2(g) → H2O(l) ). What is the standard enthalpy of reaction for the following reaction: H2O(l) + ½ O2(g) → H2O2(l)?

Benzene is a colorless liquid, its boiling point is at 80oC, and it is classified as an aromatic hydrocarbon. What is the chemical formula for benzene?

(A) is benzene; (B) is phenol; (C) is an ether; and (D) is a carboxylic acid. Benzene is an aromatic, ringed compound containing six carbons.

Organisms can be categorized into one of the following two groups: autotroph - an organism that obtains energy from sunlight

(in the case of green plants) or inorganic compounds (in the case of sulfur bacteria) ; heterotroph - an organism that use organic

An eye specialist prescribes spectacles with a combination of a convex lens of focal length 40 cm in contact with a concave lens of focal length 25 cm. What is the power of that lens combination? +1.5 D -1.5 D +6.67 D -6.67 D

-1.5 D Explanation: The power of the lens combination is P=P1+P2= 1/f1+1/f2 = 1/0.4+1/(-0.25) = -1.5m-1 = -1.5 D.

The chemical equation for burning methanol is: CH3OH (l) + 3/2 O2 (g) = CO2 (g) + 2 H2O (l). Calculate the heat of reaction (in kJ) for combustion of methanol given the following heats of formation, all in KJ/mole. Heats of formation methanol (liquid): - 238.6; carbon dioxide (gas): - 393.5; water, and (liquid): - 285.8.

-726.5 The heat of reaction is calculated as the sum of the heats of formation of the products, minus the sum of the heats of formation of the reactants. The heat of formation of oxygen in its natural state (O2 gas) is zero by definition. The other heats of formation are given. The calculation is as follows: ΔH = -393.5 + 2(-285.8) - (-238.6) = -726.5 Kj

A mitochondrion (labeled # 9 in the diagram) is defined as: The correct answer is (D) "Powerhouse" of the cell

. The nucleolus is a small structure in the nucleus that helps in the formation of ribosomes and the nucleus is the "brain" of the cell.

What is the formula for hydrochloric acid, a component of gastric juices? Both hydrogen and chloride only have a single charge when they are in their ionic form

. They are both monoatomic ions and bind together in a 1:1 ratio thus the molecular formula for hydrochloric acid is HCl.

Absolute zero is the lowest temperature that it is possible to reach. In terms of the Kelvin scale, this is

0 Absolute zero is the point at which all atomic action apparently stops. On the Kelvin scale, which is a thermodynamic temperature scale, this is 0

When 0.243 grams of Magnesium is reacted, how much hydrogen will be produced? Mg(s) + 2HCl(aq) à MgCl2(aq) + H2(g). 0.02 grams. 0.20 grams. 2.00 grams. 0.01 grams.

0.02 grams. Explanation: If you react 0.243 grams of magnesium, it will become 0.01 moles of magnesium. (0.243g/24.3g/mol). For every mole of magnesium reacted, a hydrogen mole is produced. Hence, 0.01 moles of H2 are generated, and are equivalent to 0.02 grams of hydrogen (0.01 mol x 2.0 g/mol).

The speed of light in vacuum is c. If a special glass has a refractive index of 2.5, what is the speed of light while traveling within the glass? 2.5c 0.4c c 1.25c

0.4c The refractive index of a medium is a measure of how much the speed of light is reduced while it travels inside the medium. As the refractive index of the glass is 2.5 light travels at a speed c/2.5=0.4c

The figure shows three cells of 1.5 V each and a resistor R of 3 ohm. What is the current flowing through R? 0.5 A 2 A 1.5 A 3 A

0.5 A As the cells of 1.5 V are connected in parallel with the resistor, that voltage across the resistor is 1.5 V. The current is equal to V/R or here 1.5/3 = 0.5 A.

What is the normality of a 0.25M solution of sulfuric acid (H2SO4)? 0.125N. 0.25N. 0.5N. 1.0N.

0.5N. The number of available hydrogen atoms in a molecule of sulfuric acid is X = 2. In general, normality = X times molarity. In the example, normality = (2)(0.25) = 0.5N

Talc is easily scratched by a fingernail. Which of the following Mohs scale ratings represent talc? 7 3 10 1

1 Explanation: Talc is a soft mineral which is a one reason that talc is easily scratched by a fingernail. Based on the Mohs scale, which has ratings from 1 to 10, Talc is rated as a 1. The higher the number on the Mohs scale, the higher the level of hardness measured. Hardness means how easily a mineral is scratched. So, if a miner is a 10 such as a diamond, then a 10 on the Mohs scale indicates that a diamond is hard as it requires a cutting tool to scratch it.

What is the systematic name of urea?

1-aminomethanamide

What will be the percentage error if the calculated value is 5.5 and the actual value is 5.6?

1.78% % error = (actual value -measured value/actual value) x 100 % error = (5.6-5.5/5.6) x 100 % error = 1.78%

The figure shows the path of a roller coaster that starts from a height of 40 m. The car must go over two hills, with heights of 45 and 30 m, respectively. What should be the minimum initial velocity of the car if friction is neglected? . Use g = 10 m/s2.

10 ms-1 The cart has to ascend 5m from its starting position and must have enough kinetic energy to do so. Equating kinetic to potential energy gives: m v2/2 = mgh, or v2 = 2gh = 2*10m/s2*5m, so v = √100 = 10 m/s

50 C converts to what number of degrees Fahrenheit? 106 F. 122 F. 323 F. 509 F.

122 F. Explanation: The formula for converting degrees Celsius to degrees Fahrenheit is F = 1.8(C) + 32. Thus 50 C = (1.8)(50) + 32 = 122 F.

If the distance between the Earth and the Sun were half its present value, what would be the number of days in a year? 64.5 129 182.5 730

129 Explanation: Take T1 as the time period now, and T2 as the time period when the distance is halved. According to Kepler's Law of Periods T1/T2= (R1/R2)(3/2)=(R1/(R1/2))(3/2)=2(3/2)=2√2. Therefore, T2=T1/(2√2)=365/(2√2)≈129 days.

This graph shows algae levels in Cobbossee Lake, Maine. Maine defines a nuisance algal bloom as a minimum Secchi Disc Transparency of less than 2.0 meters in lakes. Based in this definition, when was the last algal bloom? 1996 1977 1982 1985

1996 On the graph, every point below the red line indicates a nuisance algal bloom. The last year shown with a value below the red line (2.0) was 1996.

In what century did the term scientific begin being used? 16th Century. 3rd Century. 17th Century. 19th Century.

19th Century Not until the 19th Century was the term scientific generally used. Before that time, the terms experimental method and the method of science were used.

If a stationary wave is established with a string 4 m long, fixed at both ends, and vibrating in 4 segments, what is the wavelength?

2 For 4 segments l = 4 λ/2 λ = 2 x 4 /4 λ = 2

Calculate the pH of gastric juice in our stomach, provided the hydrogen ion concentration is 0.01 mole/litre.

2 pH = -log 10-2 = 2.

An object in medium M is transmitting a wave which travels with a velocity of 10m/s. It has a frequency of 100s-1. An observer notices that the frequency of the wave is 125s-1. At what velocity is the object approaching the observer? -2 m/s 12.5 m/s 2 m/s -8 m/s

2 m/s In the given question, we can use the following expression f=f0v/(v+vs) where f0 is the frequency of emitted waves, f is the frequency as measured by the observer, v is the velocity of the wave in the medium, and vs is the velocity of the source. Here f=125s-1, f0=100s-1, v=10m/s. Substituting in the expression given ,125= 100x10/(10+vs), vs=(100x10/125)-10= -2. This indicates that the source is moving towards the observer with a velocity of 2 m/s.

What is the standard deviation depicted in this graph? 40 80 20 10

20 Standard deviation is a measurement of variability that is used in statistics and probability theory. It shows how much variation there is from the average or mean. In this graph, the red lines above and below the mean indicate the standard deviation.

The coefficient of friction for each body with a horizontal plate is 0.6. What is the string's tension force between the two boxes?

20 (N) Because Mg=20<fx=24=>system is static then T=Mg=20

Based on these graphs, what year had the highest pro-ecological sentiment in this area? (Assume the same number of people respond each year. ) 2004 2000 2008 It is impossible to tell from the data given.

2000 This graph is based on surveys conducted in Waikato, New Zealand. The year with the highest number of pro-ecological responders was 2000. Pro ecological responders decreased in both the 2004 and 2008 surveys, and mid ecological responders increased at each survey. Overall the trend is a shift toward mid-ecological sentiment in this area.

For a certain gas the ratio of specific heat is 3/2. What is the value of Cp for it?

3R Cp/Cv = 3/2 = Cv + R/Cv = 3/2 This gives Cv = 2R and hence Cp = 3R.

How many rings are found in steroid compounds including cholesterol?

4 While steroid molecules can have a number of different functional groups, the core of the molecule (sterane core) will be the same across all molecules. A sterane core is shown in the figure. Steroids are involved in many biological processes including stress reactions, sexual development, and cell membrane development.

Which equilibrium reaction can be described as an acid-base reaction using the Lewis acid-base definition, but not using the Bronsted and Lowry concept?

4NH3 + Cu2+ ↔ [Cu(NH3)4]2+ In the above reaction, NH3 is donor and Cu2+ ion is acceptor. Hence, NH3 is base and Cu2+ ion is Lewis acid. Cu2+ is not acid according to Bronsted concept because it is not a proton donor.

Which of the following best represents the altitude of the cloud in the image above? 1000 m 7000 m 1900 m 5000 m

5000 m Explanation: The correct answer is 5000 m The altocumulus cloud forms between 2000 and 6000 meters. This type of cloud is made of ice crystals and water droplets from the colder temperatures that are typical for this altitude range.

The heat of reaction for combustion of methane which is the primary component of natural gas is - 890.2 kJ. The reaction is CH4 (g) + 2 O2 (g) = CO2 (g) + 2 H2O (l). Based on this information, estimate the amount of heat in kJ given off by burning 1 kg of methane. 55.5. 55,500. 89,020. 890,200.

55,500. Explanation: For each mole of methane burned, 890.2 kJ is given off. The molecular weight of CH4 is 12.01 + 4(1.008) = 16.042. Thus, the heat given off per gram of methane is 890.2/16.042 = 55.5 kJ; and the heat given off per kg of methane is (55.5 kJ/g)(1000 g/kg) = 55,500 kJ.

What is the combined resistance of a system having two resistors arranged in a parallel configuration and one resistor arranged in a series configuration? Each one has a resistance of 4Ω.

6 Ω The two resistors in parallel have a combined resistance of 2Ω. Adding the third resistor in series gives us 6Ω.

Identify the characteristics of science. A. It is used to explain the natural world. B. It is useful for testing theories about supernatural activities. C. It makes use of opinion surveys. D. It makes use of testable ideas.

A and D only Science focuses on the natural world. It does not deal with the supernatural. Also, it makes use of testable ideas and evidence, not opinions.

Define "mutation"

A change in genetic information of a cell or virus

One non-characteristic feature of a geological subduction zone would be which of the following?

A deep sea ridge is usually found along spreading zones, not near subduction zones. The Mid-Atlantic Ridge is a good example.

Helen noticed that bridges had more ice on them than regular roads during the winter. She formulated a suggested explanation that it was because the road got colder there than elsewhere, and decided to slow down when crossing bridges. This reasonable proposal for why bridges are more icy than roads is an example of which of the following?

A hypothesis A suggested explanation or reasonable proposal for predicting causal correlations for a phenomenon is called a hypothesis. A hypothesis is a working explanation that can be tested using experimental procedures to determine its validity.

What is the meaning of the acronym LASER?

A laser is a device that emits light (electromagnetic radiation) through a process called stimulated emission. The term laser is an acronym for Light Amplification by Stimulated Emission of Radiation. Lasers have uses in various fields now like optical storage, surgery, defense, cosmetics, etc.

Insulin is a peptide hormone of 51 amino acids. A person with Type I Diabetes Mellitus is found to have insulin that only has 33 amino acids. The person's insulin receptors are normal. What type of mutation certainly caused this disease?

A loss-of-function mutation is any mutation that causes a disruption in a protein's activity. In this case we do not know if it was a missense, frameshift, or point mutation that caused the peptide to be abnormally short. We do know that insulin is no longer an effective signal for cells to take up glucose.

What does this symbol represent in electrical circuits?

A photoresistor, light-dependent resistor or cadmium sulfide (CdS) cell, is a resistor whose resistance decreases with increasing incident light intensity. It can also be referenced as a photoconductor. The photoresistor is represented in electrical circuits as shown in the figure.

What is an allosteric site?

A receptor site on an enzyme molecule

What type of virus is Human Immunodeficiency Virus (HIV)?

A retrovirus HIV (human immunodeficiency virus) is a retrovirus that cuases AIDS (acquired immunodeficiency syndrome). It is difficult to treat due to the high mutability of retroviruses. Many new treatments for the HIV virus work to inhibit the reverse transcriptase enzyme.

What is/are (a) homeostasis?

A steady state, or state of balance

What is/are (a) homeostasis? Two of the same allele at a gene locus on a chromosome. A steady state, or state of balance. The evolution of a bipedal hominid. A receptor on the genome that tells the DNA to stop replication.

A steady state, or state of balance Homeostasis refers to a steady state, or in the case of an animal, internal balance. Animals maintain homeostasis in their internal environments using receptors that detect changes in the internal environment, a control center that processes this information, and an effector, which responds.

Which of the following is an example of a competitive relationship? Mycorrhizae fungi grows on plant roots to help the roots absorb water, and the mycorrhizae gets sugar from the plant. A vegetable garden is overrun with weeds. A lion eats a gazelle. A puppy gets worms.

A vegetable garden is overrun with weeds. Both organisms suffer in a competitive relationship. In this example, food, space, and water are in limited supply, so the weeds and vegetables are all struggling to survive.

How should a voltmeter be connected in an electric circuit when it is being used to measure the voltage across a particular component?

A voltmeter is used to measure voltage across a component of a circuit. It has to be connected in parallel to only that component as the voltage drop across different components of a circuit is not the same.

Cellular respiration converts oxygen and simple sugars to energy which is then stored in the cell. What is the primary energy currency of the cell?

ATP or adenosine triphosphate is what the cell uses as its energy currency.Once produced, ATP can be converted back to energy by the cell at any time.

Which of the following best explains the difference between accuracy and precision?

Accuracy is the degree of closeness to a true value; and precision is the repeatability of the measurement.

Which of the following best explains the difference between accuracy and precision? Accuracy and precision are basically the same thing. Accuracy is the repeatability of a measurement; and precision is its closeness to true value. Accuracy is the degree of closeness to a true value; and precision is the repeatability of the measurement. Accuracy refers to how accurate you are, and precision refers to how precise you are.

Accuracy is the degree of closeness to a true value; and precision is the repeatability of the measurement. Accuracy is the degree of closeness of a measured quantity to its actual (true) value. Precision is the reproducibility or repeatability to which further measurements or calculations show the same or similar results. Both are important to scientific methodology.

In the Krebs cycle, pyruvate from glycolysis is converted into what upon entering the mitochondrion?

Acetyl CoA In the Krebs cycle, pyruvate from glycolysis is converted into acetyl CoA upon entering the mitochondrion. This is accomplished by a multi-enzyme complex. Acetyl CoA will feed its acetate into the Krebs cycle for oxidation.

Horse latitudes are created from the sinking air near latitudes 30 degrees north and 30 degrees south where the trade winds form high areas of pressure.

Additionally, horse latitudes have a belt of weak surface winds. Deserts, such as the Sahara desert, is located under this high pressure horse latitude area.

Which of the following statements about viruses is NOT true?

After entering a cell, a virus immediately begins producing more viruses. Some eukaryotic viruses, such as herpes, can have a dormant phase, or resting period, within the host cell.

A meteor of mass M breaks up into two parts. The mass of one part is m. For a given separation r, when will the mutual gravitational force between the two parts be maximum?

After the meteor has broken up, the two parts weigh m and (M-m).The gravitational force between the two parts is F= G(M-m)m/r2 = (G/r2)(Mm-m2). F will be maximum when dF/dm=0 and d2F/dm2 is negative. Now dF/dm = (G/r2)(M-2m). Setting dF/dm=0 we get M-2m=0 or m=M/2. Also, when m=M/2, d2F/dm2= -2G/r2 which is negative.

Which of the following best describes the respiration of a frog?

Air/water, lungs/skin, positive pressure breathing For most terrestrial animals, air is the respiratory medium. Frogs can also exchange gases via water across their moist skin. A frog uses positive pressure breathing in which it lowers its jaw, expands its mouth, raises the jaw and pushes air into the lungs.

Alfred Wegener is famous for which geological theory? Correct answer: The Continental Drift Theory

Alfred Wegener was the first person to publicly note how the Continents seemed to fit together. This led to the Theory of Continental Drift, an idea not well received at the time.

Seaweeds are what? Algae. Plants. Animals. Symbionts.

Algae Seaweeds are algae. They have the most complex multicellular anatomy of all algae. Some have differentiated tissues and organs that are usually associated with plants. Seaweeds are an important source of food and food products for humans.

Photosynthetic protists are what? Algae. Archaezoa. Dinoflagellates. Protozoa.

Algae There are three general categories of protists. Photosynthetic, plant like protists are called algae. The term algae has no basis in phylogeny and no significance in taxonomy. Algae are simple aquatic photoautotrophs that include some organisms that are sometimes classified in the plant kingdom.

Why do all the elements of Group I have similar reactivity?

All of the elements in Group I, the Alkali Metals, have one valence electron. This electron is easily given up in a chemical reaction, thus they become +1 ions

As waters would rise and fall, green algae would be deposited on dry land along coastal areas. Those organisms that developed jacketed reproductive organs could survive and thrive without water, thus giving rise to land-based plants.

All other answer choices are examples that support the green algae-plant evolution hypothesis. The correct answer therefore, is (B).

Which of the following behaviors decreases the individual's fitness while increasing the recipient's fitness? Altruism Most behaviors evolve due to the fact that they benefit, or increase the fitness of the individual.

Altruism is a behavior that reduces the individual's fitness and increases the fitness of the recipient. Individuals may be more prone to engage in altruistic behavior toward closely related individuals.

Which of the following best describes H2PO4-?

Ampholyte Intermediate forms of polyprotic acids are ampholytic, because they can both accept and donate protons.

The chemical definition of an alcohol is which of the following?

An organic compound with a hydroxyl group. In chemical terms, an alcohol is any organic molecule in which a hydroxyl group (-OH) is bound to a carbon atom. Ethanol is one of many alcohols of the form CnH2n+1OH.

A black bug that has bright yellow triangles on its back and lives on a plant that has light green leaves and white flowers probably has what type of coloration?

An organism that has a conspicuous coloring and is distasteful or dangerous to eat has aposematic coloration. An organism that has cryptic coloration will blend with its background in order not to be noticed. Disruptive coloration means that some parts of the organism's body blend into the background, while other parts starkly contrast with the background, making it unrecognizable. When an organism has a light underbelly and a darker back, it is countershaded. When the organism is in the light and the underbelly is shaded, it looks flat and unrecognizable. The correct answer therefore, is (B).

Explanation: The correct answer is: rhyolitic magma. Basaltic magma has very low viscosity. It flows freely because it has low silica content. viscosity as much as the silica content does.

Andesitic magma is found in oceanic crust and sediment. It is 50 - 60 % silica, so it has intermediate viscosity. Magma often contains feldspar (although the term "feldsparic magma" is typically not used); the amount of it doesn't affect the

Which statement is NOT true about angular momentum?

Angular momentum (also known as moment of momentum) of a particle about a given origin is a vector defined as: L=r x p, in which L is the angular momentum of the particle, r is the position vector of the particle, p is the linear momentum of the particle, and x is the vector cross product. It has SI units of kg m2/s.

Homeostasis refers to a steady state, or in the case of an animal, internal balance.

Animals maintain homeostasis in their internal environments using receptors that detect changes in the internal environment, a control center that processes this information, and an effector, which responds.

What is an autotroph? An organism that gets energy from sunlight

Animals require energy to support the processes of life: movement, foraging, digestion, reproduction, growth, work. materials as a source of energy.

Who is widely regarded as the "Father of modern chemistry"?

Antoine-Laurent de Lavoisier While all of the men listed are giants in the field of chemistry, Antoine-Laurent de Lavoisier is generally accepted to be the father of modern chemistry. Among his many accomplishments was elucidation of the law of conservation of mass.

Identify the dependent variable in an experiment determining the effect of a skin cream on people with a skin rash.

Any measure of the effectiveness of the treatment, such as the first answer would be a dependent variable. The other three are examples of independent variables.

Cellular senescence is the process by which cells naturally die after a certain period of time. The process by which cells undergo a purposeful and programmed death is called what?

Apoptosis is the name given to programmed cell death. A cell might execute its apoptotic program (akin to a self-destruct program) if there is some sort of damage to the cell's DNA. Apoptosis is useful to prevent the uncontrolled cell division that is seen in cancerous tumor growth.

If the sun stopped shining suddenly, approximately how long would it take to see the light go out?

Approximately eight (8) minutes and 18 seconds, which is the distance that the sun is from Earth-- 8.3168 light minutes away.

Heat is applied to a pot of water until it begins to boil and generates steam. What happens to the water temperature during this process?

As heat is applied to water that is below its boiling point, its temperature will increase until it reaches the boiling point of water. The heat will then be used to vaporize the liquid water, and the temperature will remain at the boiling point.

Malaria is a member of which of the following phylum? sporozoans Malaria is one of the world's most serious infectious diseases

As many as 2 million people still die from malaria every year. The sporozoan Plasmodium, which causes malaria, is carried by the female Anopheles mosquito.

pumping blood through the entire vasculature. Blood enters into the left atrium, passes across the mitral valve and collects in a relaxed left ventricle.

As pressure in the left ventricle increases due to muscular contraction, the mitral valve closes prohibiting back flow. At the same time the aortic valve opens, allowing blood to be forced into the

Methanogens are able to reduce carbon dioxide to methane using hydrogen. They are common in marshes and produce the methane found in "marsh gas." Methanogens are also strict anaerobes. Which of the following is true in relation to methanogens?

As strict anaerobes, methanogens are among the strictest. Those bacteria are poisoned by oxygen. They can exist in deoxygenated water, and in the guts of animals where they aid in digestion. Methanogens have been used in sewage treatment facilities to aid in the decomposition of solid waste. The correct answer therefore, is (A).

Which power generation system offers the least negative environmental consequences?

Aside from its aesthetic consideration, wind power generation seems to be the least environmentally destructive. To be certain, one needs to assess what it takes to build windmills, and also how that is likely to affect the environment.

What phylum would the organism pictured belong to? Phylum Chordata includes invertebrate as well as vertebrates. Chordata are bilaterally symmetrical. All have some level of segmentation and have an internal skeleton. Characteristics of Chordata include (1) a dorsal hollow nerve cord (as opposed to a ventral nerve cord) above the notochord.

At the anterior end, the nerve cord enlarges to form the brain. All chordates also have (2) a flexible, rod-like notochord which is above the digestive tract and provides internal support for the body. In vertebrates, the embryo has a notochord which is replaced by the vertebral column or backbone in an adult. All chordates also possess (3) a tail that extends past the anus, and (4) gill pouches that are present at some stage of the life cycle.

The average ocean depth on Earth is about which of the distances listed below?

Averaging the deepest points which are about 10,000 meters, and the shallow continental shelf areas, the average depth of the ocean is about 4000 meters. This is equal to the grade difference of about 12,000 feet, or the distance from sea level to the top of a small mountain on land.

Calculate the magnetic dipole filed from a current-carrying loop where the loop radius is (a) and (I) indicates the current.

B = [(μ0Ia2)/{2(a2 + z2)3/2}] ẑ Magnetic dipole field from a current carrying loop can be calculated with as B = [(μ0Ia2)/{2(a2 + z2)3/2}] ẑ.

What is the chemical formula for calcium bicarbonate?

Balancing the ions calcium Ca+2 and bicarbonate ( HCO3 -1 ), one would need to have two bicarbonates for every calcium atom. The correct formula therefore, is Ca(HCO3)2.

What is the element with the greatest atomic radius within Group 2 on the Periodic Table? Beryllium. Magnesium. Calcium. Barium.

Barium. Atomic size increases as you move down through a group of elements. Each row has one more electron energy shell. Barium therefore, is the largest atom.

Why is it so important to manage wetland area properly?

Because wetlands filter drinking water and prevent flooding. Wetlands help by filtering drinking water and they help to prevent flooding. Wetlands are also places where life is diverse, thus those healthy areas need to be protected.

What is the product obtained when Ethyne is passed through a red hot tube at 873K?

Benzene Ethyne on passing through red hot iron tube at 873K undergoes cyclic polymerization to form benzene.

What will happen as bond order increases?

Bond lend decreases and bond strength increases Bond length decreases with the increase of bond order. Bond lengths of c-c and c=c and c=-c are, 154pm, 133pm, and 120pm respectively. This is due to the reason that %age of s-character in hybridized orbital of c-atoms increases from c-c to c=c and c=-c.

A mineralized connective tissue formed by osteoblasts is what? Correct answer: Bone

Bone is a mineralized connective tissue that is formed by osteoblasts. The osteoblasts deposit a matrix of collagen, calcium, magnesium, and phosphate ions.

Which of the following is NOT a function of the molecules norepinephrine and epinephrine? Heart rate is slowed Epinephrine and norephinephrine molecules are synthesized from tyrosine in the adrenal medulla.

Both substances are secreted in response to stress. They increase metabolic rate, increase the rate of glycogen breakdown, stimulate the release of fatty acids from fat cells and increase the rate and stroke volume in the heart.

Identify the bond necessary for organometallic compounds.

By definition, organic compounds always contain carbon. Organometallic compounds may not contain more than one metal, but will always contain a carbon to metal bond; and the metal will not be bound through hydrogen.

The heat of reaction for C3H8(g) + 5O2(g) → 3CO2(g) + 4H2O(l), is -2220 kJ/mole. Which of the following statements are true for this reaction?

By definition, the heat of reaction is negative for an exothermic reaction. That makes A true and C false. This is indeed a combustion reaction, so B is true. As an exothermic reaction, heat will be given off to the surroundings, thus increasing the energy level of the surroundings, so statement D is true.

Galileo is most generally credited with being the father of the scientific method.

By developing the tools of observation, hypothesis, mathematical deduction and confirmatory experiment, he founded the scientific methodology that we still use today

All of the following molecules are hydrophobic EXCEPT:

C6H12O6 Any molecule with only one kind of element with be nonpolar, which means it will be hydrophobic. C6H12O6 is the glucose molecule, and it contains carbon and hydrogen as well as oxygen, and oxygen aways forms a polar convalent bond with other atoms. Being polar means the molecule will have a small charge, so the molecule is hydrophilic.

Which organic compound has the highest reactivity in a SN2 reaction?

CH3CH2CH2CH2Br The reactivity in SN2 reactions follows the reverse order of SN1 as the steric hindrance around the electrophilic carbon increases with addition of CH3 group.Hence, (CH3)3CBr will be least reactive and CH3CH2CH2CH2Br will be most reactive. Hence, more the number of -CH3 group less the reactivity towards SN2 reactions.

Identify the term for the ability of a buffer solution to resist pH change when an acid or base is added.

Capacity The extent of which a buffer can resist change in pH is known as buffer capacity.

What form(s) the skeletons of most organic molecules?

Carbon chains form the skeletons of most organic molecules. It is the variation in carbon skeletons that contributes to the diversity of most organic molecules.

Kelly observed strange beads of dirt with small holes next to them on his front lawn after every rain. What kind of scientific investigation could he conduct to find out what formed those holes and beads?

Carefully observe the lawn for a long, continuous period after a rain. Carefully observing the lawn would be the most scientific way to answer his question. All the other methods involve research only.

Which law explains pressure dependence on temperature under constant volume (isochoric process)?

Charles's law According to this law pressure of constant amount of gas, at constant volume increases linear with increasing temperature. This dependence is graphically represented with straight vertical lines, named isochors

Matter can exist as a solid, liquid, or gas and it can be converted from one form to another. Which of these four are examples of chemical change?

Chemical changes form new substances with properties that are different from those of the original substances.

How does a chemoheterotroph obtain energy and carbon? From organic sources only

Chemoheterotrophs must consume organic molecules for both energy and carbon. This type of nutritional mechanism is found commonly in fungi, protists, prokaryotes, some plants and animals.

Mount Pinatubo located in the Philippines is an example of which of the following?

Circum-Pacific Belt is known as the Pacific Ring of Fire. This type of belt identifies the circumference location of the belt that surrounds the Pacific Plate. Volcanoes such as the one in Mount Pinatubo located in the Philippines is in the Circum-Pacific Belt.

The network of membranous tubules and sacs that make up the endoplasmic reticulum are called what?

Cisternae The endoplasmic reticulum is made up of a complex network of membranous tubules and sacs called cisternae. The membrane separates the internal compartment of the endoplasmic reticulum from the cytosol.

The network of membranous tubules and sacs that make up the endoplasmic reticulum are called what? Microtubules. Vacuoles. Cisternae. ER envelope.

Cisternae The endoplasmic reticulum is made up of a complex network of membranous tubules and sacs called cisternae. The membrane separates the internal compartment of the endoplasmic reticulum from the cytosol.

Which of the following is NOT something spiders use their silk for?

Cocoon formation for pupae

Which of the following is NOT something spiders use their silk for? Catching food. Cocoon formation for pupae. Escape from danger. Egg covering.

Cocoon formation for pupae Spiders have the unique adaptation of spinning silk. This silk is spun by organs called spinnerets. The silk is a protein that is in liquid form in special abdominal glands and solidifies into fibers when spun. The silk is used for more than just spinning webs to catch food. Spiders also use the silk as drop lines for rapid escape from danger, as egg covers, and even as gift wrapping for food that certain male spiders offer females during courtship. Spiders do not have a pupal stage in their development, nor do they form cocoons.

Which of the following molecules functions in the electron transport system?

Coenzyme NADP is the coenzyme that transfers hydrogen and electrons needed in dark reactions.

They favor the redundancy model, which is a community with little or no relationships between different species.

Communities generally change gradually; and by definition, must have more than one species. The correct answer therefore, is (B).

Which of the following is an example of conditioned behavior? a dog is brought to the veteranarian in a small wire crate and in the future refuses to enter the wire crate

Conditioning takes place when an animal learns an unusual behavior because of the expectation of some other event. The dog is conditioned not to enter the wire crate because it expects to return to the veterinarian. The rest of the examples are normal instinctual behaviors.

Which of the following is the most common gymnosperm? Correct answer: conifers

Conifers are by far the most common gymnosperms, with more than 500 known species. The phylum Coniferophyta includes pines, spruces, firs, cedars, sequoias, redwoods, junipers, and yews.

The following equation is the representation of a fluid flow continuity equation in a rectangular coordinate. Which condition is true for this mathematical representation?

Constant Density The following continuity equation in rectangular coordinate fulfils the condition of constant density.

Elevation is defined as the distance of a location above or below sea level. On a topographical map, elevation is indicated by the use of:

Contour lines Contour lines are designed to show elevation.

Of the following, explosive eruptions are a characteristic of?

Convergent volcanism Convergent volcanism occurs when tectonic plates collide at convergent boundaries forming subduction zones. Further, oceanic crust descends into the mantle forming magma, which then mixes with rock, minerals and sediment causing an explosive eruption.

Which of the following proteins helps pinch animal cells in two during cell division? microtubules intermediate filaments cilia microfilaments

Correct Answer: microfilaments Explanation: Correct answer: microfilaments Microfilaments are the proteins that make muscle cells contract, help pinch animal cells in two during cell division, allow cells like amoebae to crawl, and act as railroad tracks for organelles in some cell types.

Plants gather the sun's energy with which of the following? chloroplasts pigments cuticles stomates

Correct Answer: pigments Explanation: Correct answer: pigments Plants gather the sun's energy with light-absorbing molecules called pigments. The plants' principle pigment is chlorophull: chlorophyll a and chlorophyll b. Chlorophyll absorbs light very well in the blue-violet and red regions of the visible spectrum, does not absorb light well in the green region of the spectrum.

Millon's test is used to detect the presence of proteins. What cause the reaction in this test?

Correct Answer: -OH group Millon's test is given by any compound containing a phenolic hydroxy group.

If the temperature of the cold reservoir is Tc and the temperature of the hot reservoir is Th, what is the maximum efficiency of an operating heat engine? 1- Tc / Th 1-Th / Tc Tc / Th Tc / Th +1

Correct Answer: 1- Tc / Th Explanation: Carnot's theorem formally states that: No engine operating between two heat reservoirs can be more efficient than a Carnot engine operating between those same reservoirs. The efficiency of a Carnot engine operating between a cold and a hot reservoir with temperatures Tc and Th respectively is 1- Tc / Th.

A telescope has an objective of focal length of 100 cm, and an eye-piece of focal length of 5 cm. What is the magnifying power of the telescope when both the object and the image are at infinity? 0.2 2.0 20 200

Correct Answer: 20 Explanation: In normal adjustment, (i.e. when both the object and the image are at infinity), the magnification M=fo/fe=100/5=20.

What is a gated ion channel? A passive channel that allows substances in and out of the cell based on the principal of osmolarity. A cell membrane function that causes cell death. A cell function that opens and closes in response to stimuli and causes a change in the membrane potential. A function of the immune system that serves as a defense mechanism for the cell.

Correct Answer: A cell function that opens and closes in response to stimuli and causes a change in the membrane potential. Explanation: Correct answer: A cell function that opens and closes in response to stimuli and causes a change in the membrane potential Ion channels are central components in a large number of biological processes that involve rapid changes in cells, such as cardiac, skeletal, and smooth muscle contraction, epithelial transport of nutrients and ions, T-cell activation and pancreatic beta-cell insulin release. Neurons and some other cells have gated ion channels. These channels open and close in response to stimuli and cause a change in the membrane potential of the cell.

Define "mutation" A change in genetic information of a cell or virus. A disfiguring of an individual. A terminal translation codon error. The combination of genetic material from two non-like organisms.

Correct Answer: A change in genetic information of a cell or virus. Explanation: Correct answer: A change in genetic information of a cell or virus A mutation is a change in the genetic information of a cell or virus. It may involve a large portion of the chromosome, or it may affect just one or a few nucleotides. If the mutation occurs in the gamete, or sex cell, it may be passed on to offspring.

According to Bernoulli's Principle, what is accompanied with an increase in speed of an ideal fluid? An increase in pressure An increase in the fluid's potential energy A decrease in pressure No change in potential energy

Correct Answer: A decrease in pressure Explanation: Bernoulli's Principle states that for a flow of a liquid without any viscosity, an increase in the speed of the fluid occurs simultaneously with a decrease in pressure or a decrease in the fluid's potential energy.

What is an allosteric site? A receptor site on an enzyme molecule. A site on an inhibitor. The active site on the enzyme. An active site that has been rendered unreceptive by an inhibitor.

Correct Answer: A receptor site on an enzyme molecule. Explanation: Correct answer: A receptor site on an enzyme molecule An allosteric site is a specific receptor site on a part of the enzyme molecule that is remote from the active site. Regulatory molecules bind to the allosteric site and change the enzyme's shape and function. This acts as a regulatory process in enzyme activity in the cell.

hat is the device that converts alternating current to direct current? A transistor An inverter A solenoid A rectifier

Correct Answer: A rectifier Explanation: A rectifier is an electrical device that converts alternating current (AC) to direct current (DC), through a process known as rectification. Rectifiers have many uses, including as components of power supplies, and as detectors of radio signals. Rectifiers may be made of solid state diodes, vacuum tube diodes, mercury arc valves, and other components.

What type of virus is Human Immunodeficiency Virus (HIV)? A provirus. A retrovirus. A temperate phage virus. A repressor virus.

Correct Answer: A retrovirus. Explanation: Correct answer: A retrovirus HIV (human immunodeficiency virus) is a retrovirus that cuases AIDS (acquired immunodeficiency syndrome). It is difficult to treat due to the high mutability of retroviruses. Many new treatments for the HIV virus work to inhibit the reverse transcriptase enzyme.

Proteins are composed of sequences of repeating units called what? Nucleotides. Base pairs. Amino acids. Carbon chains.

Correct Answer: Amino acids. Explanation: Correct answer: Amino acids Proteins are composed of repeating units called amino acids. Amino acids come in 20 different types (with names like alanine, tyrosine, glutamine), each one modeled on the same chemical plan, but structurally slightly different to one another. The order of the amino acids in protein chains is different for and unique to each and every protein.

Which of the following statements is not true with respect to an element? An element is a pure substance. An element contains either one or two types of atoms. An element cannot be decomposed further by simple chemical means. An element has a definite composition.

Correct Answer: An element contains either one or two types of atoms. Explanation: The second statement should be: "An element contains only one kind of atom" in order to be true. The other three statements are true with respect to an element.

Which statements are true for a strong base? A. A strong base will be greater than 5.0 molar in concentration. B. A strong base will ionize virtually completely in weak aqueous solution. C. A strong base will be greater than 5.0 normal in concentration. D. The normality of a strong base solution will equal [OH-].

Correct Answer: B and D only. Explanation: The term, strong base, has nothing to do with the concentration of the solution. So Statement A and Statement C are false. The other two statements for a strong base are true.

An alloy is a metal that is made up of two or more substances. Sterling silver is an alloy of silver and copper. Which of the following is also an alloy? Aluminum Lead Gold Brass

Correct Answer: Brass Explanation: The correct answer is (D). Aluminum, lead, and gold are all chemical elements in their own right; brass is an alloy made up of the chemical elements of copper and zinc.

Which of the following best describes the way that volcanoes can help build up new land? By adding heat to the Earth's surface By adding gases to the Earth's atmosphere By adding lava to the Earth's surface By adding water vapor to the Earth's atmosphere

Correct Answer: By adding lava to the Earth's surface Explanation: The correct answer is (C). Although heat, gases, and water vapor are all released by volcanoes, they do not help build up new land. By adding lava, which cools into rock, to Earth's surface, new land is created.

Which is a structural isomer of CH3CH2CH2CH2OH? CH3CH2OCH3 CH3CH2CH2OH CH3CH(OH)CH2CH3 CH3CH2CH2CH3

Correct Answer: CH3CH(OH)CH2CH3 Explanation: A structural isomer is an alternate arrangement of the atoms with the same molecular formula. Only (C) has the same molecular formula.

What is organic chemistry's main focus? Oxygen containing compounds Carbon containing compounds Proteins and enzymes Carbohydrates

Correct Answer: Carbon containing compounds Explanation: Organic chemistry is the study of carbon atoms and carbon compounds. There are about ten different groups of carbon compounds with different properties and uses. Some of them are alkanes, alcohols, ethers, carboxylic acids, aldehydes, ketones. esters, and amines

What is the most important type of plastid in a plant cell? Vacuole. Golgi body. Chloroplast. Tonoplast.

Correct Answer: Chloroplast. Explanation: Correct answer: Chloroplast The chloroplast is the most important plastid in the plant cell. Photosynthesis is carried out in the chloroplast. In photosynthesis, sunlight is converted to chemical energy stored in sugars.

Which of the following terms is most often used to describe trees that shed their leaves seasonally? Decadent Dormant Deciduous Definite

Correct Answer: Deciduous Explanation: The correct answer is (C). Deciduous is a term that literally means "tends to fall off," and is most used when describing trees that shed their leaves during autumn, such as Maple and Elm trees.

What other source of energy besides gasoline do commonly produced hybrid automobiles use? Solar energy Electricity Wind energy Hybrid cars do not use gasoline at all

Correct Answer: Electricity Explanation: Most automobile manufacturers have at least one type of hybrid vehicle. In addition to gasoline internal combustion, hybrid cars and trucks also use an electric motor. Fully electric cars have been available for decades yet they have not received widespread support.

The adrenal glands are part of what organ system? Circulatory system. Endocrine system. Excretory system. Lymphatic system

Correct Answer: Endocrine system. Explanation: The adrenal glands are located just above the kidneys, provide adrenaline--a very powerful hormone; and are part of the endocrine or hormone system.

In weather, what is a cold front? Fast moving warm air runs into slow moving cold air. Slow moving cold air runs into fast moving warm air. Slow moving warm air runs into fast moving cold air. Fast moving cold air runs into a slow mass of warm air.

Correct Answer: Fast moving cold air runs into a slow mass of warm air. Explanation: In a weather pattern, a cold front is fast moving cold air running into a slow mass of warm air. It usually produces very high winds as the dense cold air settles, and pushes the lighter warm air out.

Which of the following energy sources is most likely to be abundant in California due to its position on a plate boundary? Wind Nuclear Solar Geothermal

Correct Answer: Geothermal Explanation: The correct answer is (D). A plate boundary is literally a "crack" in the plate surface which allows geothermal heat, or "heat from the Earth," to emanate from the magma 50,000 to 60,000 miles beneath the Earth's surface. California's location on a plate boundary makes it a likely location for geothermal energy.

Identify a type of natural fiber. Cellulose fiber Fiber glass Hemp Polyamide nylon

Correct Answer: Hemp Hemp is a type of natural fiber that is produced by plants and is used to make papers, biodegradable plastics, health food and fuel.

Which of the following makes an idea more scientifically acceptable? No one disputing the idea in a scientific forum Hypotheses are made related to the idea Alternative hypotheses are proposed and confirmed Increasing scientific evidence supports the idea

Correct Answer: Increasing scientific evidence supports the idea Explanation: The correct answer is (D). The main way that an idea becomes scientifically acceptable is through increasing scientific evidence that supports the idea.

Which of the following best describes the role of the esophagus (labeled # 1 in the above diagram) in the digestion process? It aids in the absorption of nutrients from food It carries food from the mouth to the stomach It carries food from the stomach to the intestines It releases acid and mixes food

Correct Answer: It carries food from the mouth to the stomach Explanation: The correct answer is (B). The esophagus is responsible for carrying food from the mouth to the stomach.

There used to be nine planets until Pluto was downgraded to a dwarf planet. Which of the following lists includes the eight remaining planets in their order from the sun? Mercury, Venus, Earth, Mars, Jupiter, Saturn, Neptune, and Uranus. Mercury, Venus, Mars, Earth, Jupiter, Saturn, Neptune, and Uranus. Mercury, Venus, Earth, Mars, Jupiter, Saturn, Uranus, and Neptune. Mercury, Mars, Earth, Venus, Jupiter, Saturn, Neptune, and Uranus.

Correct Answer: Mercury, Venus, Earth, Mars, Jupiter, Saturn, Uranus, and Neptune. Explanation: The correct order of the eight remaining planets is Mercury, Venus, Earth, Mars, Jupiter, Saturn, Uranus and Neptune.

The following picture illustrates what physical principle? Conservation of angular momentum. An inelastic collision. Newton's First Law. Newton's Third Law.

Correct Answer: Newton's Third Law. Explanation: Newton's Third Law is illustrated by the photo. The simplest way of stating that Law is that for every action there is an equal and opposite reaction.

A reaction A(g) + B(g) → 2C(g) is in equilibrium at a certain temperature. Can the amount of products be increased by adding a catalyst and can the amount of products be increased by increasing pressure?

Correct Answer: No, No i) No, because catalyst does not disturb the equilibrium point. ii) No, because np = nr

The idea that despite new exploration and development projects, modern society has reached its maximum supply of petroleum is called which of the following? Oil maximum. Supply side energy economics. Peak oil. Maximum oil production.

Correct Answer: Peak oil. Explanation: The term used is "peak oil", and it means that we have reached the peak in the oil curve being supplied to the world. For an oil-dependent society, the consequences of reaching that point are very important.

A rating system that is used to measure the energy released from a seismic wave is called aggregate rating density rating Richter scale rating deflation rating

Correct Answer: Richter scale rating Explanation: Richter scale is a numerical rating that measures the energy released during an earthquake. Further, the Richter scale measures the energy of the largest seismic waves during the earthquake which gives us the magnitude. Also, the Richter scale uses amplitude to determine the size of the seismic wave.

Which of the following would best explain why a day on Saturn only lasts about ten Earth hours? Saturn is much farther from the Sun than Earth Saturn's orbit has greater eccentricity than Earth's Saturn rotates more rapidly than Earth Saturn is less dense than Earth

Correct Answer: Saturn rotates more rapidly than Earth Explanation: The correct answer is (C). A planet's rotation is what determines the length of its days. Since Saturn rotates faster than Earth, each day is only ten hours long as opposed to twenty-four.

The study of rock layers and how they relate to one another is which of the following? Stratigraphy. Stratilology Sedimentary science. Stratification science.

Correct Answer: Stratigraphy. Explanation: Stratigraphy is the study of rock layers and layering, or stratification. It is mostly used in the study of sedimentary or layered volcanic rocks.

The body functions in this diagram are all controlled by the Autonomic nervous system. Those to the right of the diagram are controlled by what nervous system? Sympathetic. Parasympathetic. Basal autonomic. Para-autonomic.

Correct Answer: Sympathetic. Explanation: Correct answer: Sympathetic The autonomic nervous system provides most organs with a two sets of nerves - the sympathetic and parasympathetic. The sympathetic system prepares the body for strenuous muscular activity, stress, or emergencies. The parasympathetic system operates during normal situations, permits digestion, and lowers activity for conservation of energy. These systems usually work in opposition to each other

Which of the following is the largest body in our solar system? The Sun Jupiter The Moon Earth

Correct Answer: The Sun Explanation: The correct answer is (A). The solar system is based on its single star, the sun, which is the largest body in our solar system.

Identify the false statement concerning a particle moving in a circle with a constant angular speed. The velocity vector is along the tangent to the circle. The acceleration vector is along the tangent to the circle. The acceleration vector points toward the center of the circle. The velocity and acceleration vectors are perpendicular to each other.

Correct Answer: The acceleration vector is along the tangent to the circle. Explanation: The question considers the case of centripetal acceleration of a particle for which the velocity vector is tangential to the circle, and the acceleration vector is directed toward the center. Options a,c and d therefore, are true. Only option b is false because if the acceleration vector were along the tangent, it would change the angular speed of the particle.

A paperclip has a positive charge. A balloon has a negative charge. What will happen when the paperclip and the balloon are brought close together? The balloon and the paperclip will be repelled by one another The balloon and the paperclip will be attracted to one another The paperclip will spin in circles The paperclip will remain in place

Correct Answer: The balloon and the paperclip will be attracted to one another Explanation: The correct answer is (B). Since opposite charges attract, the paperclip and the balloon will be attracted to one another.

When viewed from outer space, the Earth appears to be mostly blue. Which of the following explains the reason why this is the case? The majority of the Earth is covered with ice The majority of the Earth is covered with mountains The majority of the Earth is covered with water The majority of the Earth is covered with deserts

Correct Answer: The majority of the Earth is covered with water Explanation: The correct answer is (C). Over 70% of the surface of the Earth is covered by oceans, which is why the Earth appears to be mostly blue when viewed from outer space.

Identify the term that does not belong in the same category as the other terms. Ergs Calories BTU Watt

Correct Answer: Watt Explanation: Ergs, Calories and BTU are units of energy, as is the Joule. Watts are a unit of power equal to one Joule per second.

Which power generation system offers the least negative environmental consequences? Windmill power generation. Nuclear power generation. Coal plant power generation. Hydroelectric power generation.

Correct Answer: Windmill power generation. Explanation: Aside from its aesthetic consideration, wind power generation seems to be the least environmentally destructive. To be certain, one needs to assess what it takes to build windmills, and also how that is likely to affect the environment.

Which type of magma is found along oceanic-continental subduction zones? basaltic magma rhyolitic magma andesitic magma oceanic magma

Correct Answer: andesitic magma Explanation: The source material for andesitic magma is oceanic crust or oceanic sediments. Thus, this magma is found along the oceanic-continental subduction zones.

Based on the image above, what type of geologic formation seals in the deposits from petroleum? anticlines volcanic released gases photovoltaic cells solar heat

Correct Answer: anticlines Explanation: The correct answer is anticlines. Anticline are folded rocks. These rocks absorbs the petroleum in its layers. Since petroleum is a less dense material, it able to penetrate up through the impermeable layers of sedimentary rock, like limestone and sandstone.

Which of the following statements is FALSE about asexual reproduction? an individual can reproduce without involvement with another individual of that species asexual reproduction is limited to single-cell organisms asexual reproduction is sometimes used to describe reproduction in self-fertilizing species asexual reproduction is the process through which an organism creates a similar or identical copy of itself without contribution of genetic material from another individual

Correct Answer: asexual reproduction is limited to single-cell organisms Explanation: The correct answer is (B). Asexual reproduction is not limited to single-cell organisms. In fact, most plants have the ability to reproduce asexually

A type of sand dune that creates a solitary crescent shape is which of the following? transverse longitudinal parabolic barchan

Correct Answer: barchan Explanation: The correct answer is barchan. A barchan dune creates solitary crescent shapes. Further, the barchan dunes are developed from small amounts of sand and are located in areas that have small amounts of vegetation or no vegetation.

A calcium deficiency would affect which part of the body the most? eyes bones skin intestines

Correct Answer: bones Explanation: The correct answer is (B). Calcium is an extremely important component in maintaining a strong skeletal system.

Which of the following is a classification of sedimentary rock? foliated metamorphic rock clastic nonfoliated rock quartzite

Correct Answer: clastic Explanation: The correct answer is clastic. Sedimentary rocks are classified as clastic, chemical or biochemical. Foliated metamorphic rock and nonfoliated rock are incorrect answer choices as these are classifications of metamorphic rocks. Then, quartzite is an incorrect answer choice as this is a type of nonfliated metamorphic rock that is hard and lightly colored.

Scientists determined from seismometers that the moon has layered regions that are made of magma regolith crust perigee

Correct Answer: crust Explanation: The correct answer is crust. Based on data from seismometers, the moon has features that are similar to the Earth. These features are a crust, upper mantle, lower mantle and core.

Which of the following identifies a type of wind erosion? moraines till stratified drift deflation

Correct Answer: deflation Explanation: The correct answer is deflation. Wind, as a powerful contributor to erosion, picks up and carries sediment. Further, wind has two types of erosion agents which are deflation and abrasion.

Most caves are formed when groundwater evaporates dissolves limestone rejuvenates erodes bed rock

Correct Answer: dissolves limestone Explanation: The correct answer is dissolves limestone. Caves are natural underground openings with a connection to the Earth's surface. Most caves are formed in the zone of saturation when groundwater dissolves limestone. It infiltrates the cracks and joints of limestone formations, gradually dissolving the adjacent rock and enlarging passages to form a network of openings.

A gust of wind blows soil across an open field. This type of action is called intrusive rocks mitigation cementation erosion

Correct Answer: erosion Explanation: The correct answer is erosion. Erosion caused by wind is visible as the gust of wind blows soil across an open field. And erosion occurs when the force from the wind as it lifts sediment up from the ground and out of the rocks and then carries it away to other locations.

A mineral that breaks and exhibits a jagged edge is known to have a streak cleavage fracture fluorescence

Correct Answer: fracture Explanation: Minerals that break unevenly across a rough or jagged edge is called a fracture. Cleavage is an incorrect choice as minerals that have a cleavage breaks and have uneven planes. Further, fluorescence is an incorrect choice as fluorescence happens when a mineral interacts with an ultraviolet light and the light causes to mineral to glow in the dark.

Which of the following equalizes the Earth's thermal energy? eccentricity synchronous rotation universal gravitation global winds

Correct Answer: global winds Explanation: The correct answer is global winds. Global winds are created by the coriolis effect and the Earth's heat imbalance. The global winds carry colder air to warm areas located near the equator. Also, global winds carry warmer air to colder regions near the poles. Further, global wind system assist with stabilizing thermal energy on the Earth.

If a doctor describes her patient as dehydrated, she is saying that the person is contagious should be quarrantined is going to die has lost a great deal of water

Correct Answer: has lost a great deal of water Explanation: The correct answer is (D). By definition, dehydration is the deprivation or loss of water.

The work done in a particle from infinity to A is WA , to B is WB , and to C is WC. Points A and C are the end points of the major axis. What condition is true?

Correct Answer: i = iii > ii Work one is independent of path. It depends only on initial and final positions.

After working out or running, muscles generally become fatigued. Which of the following chemicals is the main cause of this? sugar oxygen amino acids lactic acid

Correct Answer: lactic acid Explanation: The correct answer is (D). The chemical that builds up in tired muscles is called lactic acid.

A meridian is a line of which of the following? altitude longitude latitude climate

Correct Answer: longitude Explanation: The correct answer is (B). A meridian line runs from north to south and refers to distance from the equator.

Which of the following is an example of nonfoliated metamorphic rocks? marble slate schist gneiss

Correct Answer: marble Explanation: The correct answer is marble. Marble and quartzite are two common types of nonfoliated rocks. Marble is created from the metamorphism of quartz rich sandstone. Slate, schist and gneiss are incorrect choices as these are examples of foliated metamorphic rocks.

Uranium has 92 protons and 146 neutrons. The total protons and neutrons represents what number? atomic mass isotopes ion

Correct Answer: mass Explanation: The mass number is the total of the protons and the neutrons. In the case of uranium, its mass number is 238 (92 protons and 146 neutrons). Atomic number is an incorrect choice as this represents the number of protons that are located in an atom's nucleus. Further isotopes have atoms that are made of the same elements but with different mass numbers. Finally, ion is an incorrect answer choice since an ion describes an atom that gains or loses an electron.

Petrified wood, as shown in the image above, is a fossil remain that has undergone what type of process? evolution original preservation mineral replacement recrystallization

Correct Answer: mineral replacement Explanation: The correct answer is mineral replacement. In the process of mineral replacement, the open pores in an organism's hard parts such as shells contain minerals from the groundwater. The groundwater that is in the hard parts replaces the original minerals with a different type of mineral. For instance, a shell that has the mineral calcite, may be replaced by silica when its hard shell comes in contact with ground water. When trees are buried in volcanic ash, the minerals dissolve from the ash and absorbs into the pores of wood, resulting in petrified wood.

Homo sapiens are defined as modern reptiles modern birds modern marine life modern humans

Correct Answer: modern humans Explanation: The correct answer is modern humans. Modern humans are the species of homo sapiens. They were well established as the ice age began.

The process of photosynthesis creates which of the following end-products? water carbon dioxide carbon monoxide oxygen

Correct Answer: oxygen Explanation: The correct answer is (D). In photosynthesis, plants take carbon dioxide and water and convert these raw materials into food, thereby producing oxygen.

Which of the following best describes the function of the rough endoplasmic reticulum? receives and transports synthesized proteins chemically processes and packages substances synthesizes lipids and carbohydrates acts as the cell's digestive system

Correct Answer: receives and transports synthesized proteins Explanation: The correct answer is (A). The rough endoplasmic reticulum is the membranous network that receives and transports synthesized proteins.

A tree that is cut down and replaced by planting a seedling is an example of cogeneration cosmic radiation renewable resource original preservation

Correct Answer: renewable resource Explanation: The correct answer is renewable resource. When individuals cut down trees and replace the trees by planting a seedling, the trees become a renewable resource. Renewable resource is any natural resource that is replaced quickly after it is used.

Which of the following magma types contains the highest silica content? rhyolitic quartz muscovite potassium feldspar

Correct Answer: rhyolitic Explanation: The correct answer is rhyolitic. Rhyolitic is a type of magma classification. Additional types of magma are basaltic and andesitic. The magma that has the highest silica content is rhyolitic at more than 66%.

Which of the following identifies procedures that are used to perform an experiment? independent variable control scientific method controlled hypothesis

Correct Answer: scientific method Explanation: The correct answer is scientific method. Scientists gather data in many ways. Also, they use similar methods to collect the data through a series of procedures known as the scientific method.

Which of the following occurs when sediment binds together? seamount is formed sedimentary rocks are formed volcanic eruptions glacier movements

Correct Answer: sedimentary rocks are formed Explanation: The correct answer is sedimentary rocks are formed. The small pieces of rocks, sediment, forms sedimentary rocks when sediment sticks together. Also, the formation of sedimentary rocks is impacted by weathering and erosion which produces sediment. Typically, sediment is moved and deposited by wind, water, glaciers and gravity.

Which of the following best explains why radiometric dating is not used to determine the absolute age of sedimentary rocks? sedimentary rocks do not have radioactive isotopes sedimentary rocks form from preexisting rocks sedimentary rocks do not erode sedimentary rocks do not have a daughter product

Correct Answer: sedimentary rocks form from preexisting rocks Explanation: The correct answer is sedimentary rocks forms from preexisting rocks. Scientists have discovered that radiocarbon dating is not helpful in determining the absolute age of sedimentary rocks. The reason is sedimentary rocks are made from pre-existing rocks. Therefore, the original mineral is not present in these sedimentary rocks.

Mica is a dominant mineral for making furnace windows. What mineral group does Mica belong? silicates oxides carbonates halides

Correct Answer: silicates Explanation: Minerals are grouped by their chemical properties. The major mineral groups include silicates, sulfides, oxides, sulfates, halides, carbonates and native elements. The mineral mica belongs to the silicate group.

Which of the following releases large amounts of energy such as radiation from the sun's surface? solar pulsar solar black hole solar flares solar parallax

Correct Answer: solar flares Explanation: The correct answer is solar flares. Solar flares release particles and radiation from the sun's surface. These pieces are typically emitted through solar wind

What is the hybridization of the oxygen atom in the molecue OF2 ?

Correct Answer: sp3 H = ½ [6 + 2 - 0 + 0] = 8/2 = 4. Therefore, hybridization is sp3.

The principle that identifies the equal orbital and rotational periods are called umbra rotations prenumbra rotations sidereal rotations synchronous rotation

Correct Answer: synchronous rotation Explanation: The correct answer is synchronous rotation. The moon spins around the Earth once each time. As a result of this, the process of synchronous rotation is termed. With synchronous rotation, the orbital and rotational periods are equal.

The three main components to the circulatory system are the heart, the lungs and the blood vessels the heart, the blood and the blood vessels the heart, the lungs and the blood the lungs, the blood and the blood vessels

Correct Answer: the heart, the blood and the blood vessels Explanation: The correct answer is (B). Although the lungs play an important role in the circulatory process because they oxygenate the blood, the three main components of the circulatory system are the heart, the blood and the blood vessels.

The four seasons are the result of which of the following? the yearly revolution of the earth around the sun and the tilt of the earth's axis relative to the plane of revolution the yearly revolution of the moon around the earth and the tilt of the moon's axis relative to the plate of revolution the yearly revolution of the sun around the earth and the tilt of the earth's axis relative to the plane of revolution the yearly revolution of the earth around the moon and the tilt of the earth's axis relative to the plane of revolution

Correct Answer: the yearly revolution of the earth around the sun and the tilt of the earth's axis relative to the plane of revolution Explanation: The correct answer is (A). Seasons happen due to the yearly revolution of the Earth around the Sun and the tilt of the Earth's axis relative to the plane of revolution.

Sedimentary rocks from the Proterozoic time are called red beds because they contain hydrocarbons they contain iron oxide they contain carbon they contain calcite

Correct Answer: they contain iron oxide Explanation: The correct answer is they contain iron oxide. Red beds, which were sedimentary rocks that date back to the mid-Proterozoic time, contained iron oxide. It was the iron oxide that gave these sedimentary rocks the rustic red color. Also, these red beds provided evidence that the atmosphere had oxygen gas in the mid Proterozoic time.

A ball of mass m moving horizontally with a speed v collides with the bob of a simple pendulum at rest. The mass of the bob is also m. If the collision is perfectly elastic, to what height will the bob rise? v2/g v2/2g v2/4g v2/8g

Correct Answer: v2/2g Explanation: In an elastic collision between two bodies of the same mass with one of them initially at rest, the moving body is brought to rest and the other moves in the opposite direction with the same velocity. Thus, the ball will come to rest and the bob of the pendulum acquires a speed v. At this speed it will rise to a height h=v2/2g . Hence, the correct choice is b.

The physical property that describes a material's resistance to flow is called silica viscosity temperature magma flow

Correct Answer: viscosity Explanation: Vicosity describes a material's resistance to flow. Temperatue and silica content affect the viscosity of a magma. Cooler magma resists flow whereas warmer magma flows easily.

100g of water dissolves 204 g of sugar at 20ºC (293K). Which component is considered to be solvent?

Correct Answer: water Water is in the same state of matter as well as solution.

One form of asexual reproduction that plants use is called which of the following?

Correct Answer: Vegetative propagation. Vegetative propagation, also called vegetative multiplication, or vegetative cloning, is a type of asexual plant reproduction. New plants arise by growing off the parent plant without seeds or spores.

A certain trait is a sex-linked trait and the phenotype only shows up in males. If a male with the phenotype mates and produces offspring with a female that does not carry the trait, what are the chances that their male offspring will show up with the phenotype for the sex-linked trait?

Correct answer: 0% A sex-linked trait is passed to offspring by a gene on the X chromosome. Boys are XY, and a man with a sex-linked phenotype only passes on his Y chromosome, so his sons will not inherit the sex-linked trait.

Which of the following is NOT a component of muscle tissue? Contractile cells. Actin. Myosin. Chondrocytes.

Correct answer: Chondrocytes Muscle tissue is made up of long, contractile cells called muscle fibers. Muscle fibers contain actin, myofibrils and myosin.

Certain types of color blindness are a sex-linked trait. The recessive gene that determines the condition is on the x chromosome. If Andrew is color blind but his father is not, how has Andrew become color blind?

Correct answer: From his maternal grandfather's x chromosome.

A certain habitat has the following characteristics. Average daily high temperature of -5° C. Low species diversity and density. Low yearly rainfall. In which of the following biomes is the habitat located?

Correct answer: desert Not all deserts are hot- the most important characteristic of a desert is dryness

Wegener, a scientist, had which of the following backgrounds that helped him support his theory of continental drift? geochemistry geology meteorology paleontology

Correct answer: meteorology Wegener had a background in meteorology. Meteorology is the study of the atmosphere including the air that surrounds the earth. By having a background in meteorology, Wegener could identify clues about the climates based on the fossils.

Which law is used to determine if two bodies are in thermal equilibrium using the third body?

Correct answer: Zeroth law of thermodynamics Every body has a feature called temperature. Using a thermometer we can determine if two bodies are in the thermal equilibrium.

Light reactions of photosynthesis, sustain the Calvin cycle by supplying what two components?

Correct answer: ATP and NADPH

A seed is a(n) what? Embryo with its food supply contained in a protective coat. Plant sperm cell. Plant transfer cell. Ancient plant structure which evolved prior to seedless plants.

Correct answer: Embryo with its food supply contained in a protective coat. Plants with seeds evolved late in the Devonian period, around 360 million years ago. A seed is an embryo with its food supply contained in a protective coat and became an important means for plants to disperse offspring.

Which of the following would NOT affect the metabolic rate of an endotherm?

Correct answer: Initiation feedback Animals adjust their metabolic rates as conditions change. .

The reactant an enzyme acts on is referred to as the enzyme's what?

Correct answer: Substrate The reactant an enzyme acts on is referred to as the enzyme's substrate.

When discussing different types of chemical bonds, which one of the following statements is incorrect?

Covalent bonds are most common between a metal and a non-metal. Covalent bonds are most common among two non-metal elements. Ionic bonds are more common between one metal and one non-metal element.

What process do scientists use to learn the most about the Earth's interior structure?

Deflections in the paths of seismic waves from earthquakes and man-made explosions can tell the most about the unseen regions below the Earth's surface. Harder materials deflect waves differently than softer materials.

What is Danish physicist Neils Bohr most famous for? Describing the photoelectric effect. Calculating the coefficient of static and kinetic friction. Developing a more accurate model of the atom. Contributing to the development of the Heisenburg Uncertainty Principle.

Developing a more accurate model of the atom. The Bohr Model of the Atom posits that electrons travel in discrete orbits around an atom. Just like planets in a solar system, negative electrons travel in set orbits around a positively-charged nucleus.

What is Danish physicist Neils Bohr most famous for?

Developing a more accurate model of the atom. The Bohr Model of the Atom posits that electrons travel in discrete orbits around an atom. Just like planets in a solar system, negative electrons travel in set orbits around a positively-charged nucleus.

People without enough insulin in their blood can develop very serious health problems. What is one major problem that they can develop? Hypoglycemia. Toxic Shock Syndrome. Diabetes. Hepatitis.

Diabetes Lack of sufficient blood insulin and high blood sugar can lead to diabetes. Hypoglycemia occurs when the blood sugar levels are too low.

In this plant seed, the embryo is an elongated embryonic axis attached to fleshy cotyledons. What type seed is it?

Dicot In a dicot seed, the embryo is an elongated embryonic axis that is attached to a fleshy cotyledon (rudimentary leaf of the embryo). The bean plant has a dicot seed.

In this plant seed, the embryo is an elongated embryonic axis attached to fleshy cotyledons. What type seed is it? Dicot. Monocot. Coleoptile. Tricot.

Dicot In a dicot seed, the embryo is an elongated embryonic axis that is attached to a fleshy cotyledon (rudimentary leaf of the embryo). The bean plant has a dicot seed.

Which type of chemical bonding is NOT an example of non-covalent bonding?

Dipole-dipole interaction Ionic bonds, hydrogen bonds and van der Waal forces are examples of non-covalent bonds which create strong forces between atoms or molecules. Dipole-dipole interaction is an intermolecular force and is much weaker.

The selection that occurs when two extreme phenotypes are fitter than an intermediate phenotype is called known as which of the following?

Disruptive selection Occasionally, natural selection favors the two extremes. That causes alleles for intermediate forms of a trait to become less common in the gene pool.

Which property of platinum wire makes it a choice for tight seals with glass? Correct Answer: Coefficient of cubical expansion

Due to the same coefficient of expansion, change in volume is almost the same for both and hence the seal does not become loose.

Which of the following processes links together the monomers of polymers? Correct answer: dehydration synthesis

Each time two monomers link together, a water molecule comes out. Synthesizing a polymer means 'making' a polymer, and since the process loses a water molecule, the process is called dehydration synthesis.

What theories is Albert Einstein not famous for?

Einstein's name is synonomous with genius, and has many theories connected to it. The Quantum (or Heisenburg) Uncertainty Principle however, is not one.

Which of the following are mismatched?

Elephants - external gonads Production of normal sperm cannot occur at the normal body temperature of most mammals.

Which of the following are mismatched? Whale - internal gonads. Many rodents - gonads are external during breeding season and drawn back into the body at other times. Monotremes - internal gonads. Elephants - external gonads.

Elephants - external gonads Production of normal sperm cannot occur at the normal body temperature of most mammals. Many mammals testes are held outside the abdominal cavity in the scrotum to keep the temperature about two degrees Celsius below the body temperature. In many rodents the testes are drawn back into the abdominal cavity to interrupt sperm maturation between breeding seasons. Some mammals whose body temperature is low enough to allow sperm maturation, retain the testes within the abdominal cavity.

Which of the following is not associated with stress? Cortisol. Endorphins. Epinephrine. Norepinephrine.

Endorphins. Explanation: Correct answer: Endorphins In response to stress, the body secretes epinephrine, norepinephrine, cortisol and other hormones. The glucocorticoids (such as cortisol) have a catabolic action. That is, they suppress the synthesis of protein, glycogen and triglycerides. Instead, these are broken down into fatty acids, glucose and amino acids.

In which of the following are excess calories NOT stored?

Epithelial tissue When an animal takes in more calories than it needs, the excess can be stored. It is stored in the liver and muscles as glycogen, or in adipose tissue as fat if the glycogen stores are full.

The study of how animals behave in their natural environment is called what?

Ethology Ethology is the study of how animals behave in the natural environment. Ethology began during the 1930s with the work of Nikolaas Tinbergen, Konrad Lorenz and Karl von Frisch, who were joint winners of the 1973 Nobel Prize.

The study of how animals behave in their natural environment is called what? Behaviorism. Ethology. Etiology. Behavioral ecology.

Ethology Ethology is the study of how animals behave in the natural environment. Ethology began during the 1930s with the work of Nikolaas Tinbergen, Konrad Lorenz and Karl von Frisch, who were joint winners of the 1973 Nobel Prize.

Which of the following series best describes blood flow sequence through the human heart? Left atrium to mitral valve; to left ventricle; to aortic valve; to aorta

Except for deoxygenated blood heading to the lungs, the left ventricle is chiefly responsible for aorta for delivery to the tissue.

Which number will have all zeros that are significant?

Explanation: Correct Answer: 13.000 The zeros to the right of the decimal point are significant.

Which of the following elements is a diatomic gas? Helium Chlorine Mercury Xenon

Explanation: Helium is a noble gas and monatomic; chlorine is a halogen and a gas and diatomic; mercury is liquid and monatomic; and xenon is a nobel gas and monatomic. There are 7 diatomic elements: H2, N2, O2, F2, Cl2, Br2, I2. They exist as molecules of two atoms.

This growth curve is an example of what?

Exponential growth Exponential growth is growth that is ever increasing in rate. Another definition is runaway expansion, such as in population growth.

Identify the equation that represents the magnitude of the electrostatic force (F) on a charge (q1) due to the presence of a second charge (q2). (r is the distance separating the two charges and ke is the Coulomb's constant) F = keq1q2 / r F = q1q2 / ke F = ke q1q2 / r2 F = ke q2/ r2

F = ke q1q2 / r2 Explanation: According to Coulomb's law the magnitude of the electrostatic force (F) on a charge (q1) due to the presence of a second charge (q2), is given by F= ke q1q2 / r2 where r is the distance between the two charges and ke is a proportionality constant called Coulomb's constant. A positive force implies a repulsive interaction, while a negative force implies an attractive interaction. Note the similarity between this expression and the force of gravity between two masses separated by a distance r.

Which governmental agency in the United States must approve a compound as a chemotherapeutic drug before it can be prescribed to patients? EPA FBI FTC FDA

FDA Explanation: The FDA or Food and Drug Administration must approve the use of all prescription medications. This process usually takes many years from the initial safety studies to efficacy studies and beyond. Generally there are fewer than 30 new drugs approved by the FDA each year.

These cyanobacteria colonies are supposed to have figured out how to do photosynthesis BEFORE they had food to energize the process.

False answers: strobila are colonial cnidarians cyanoblocks is not a term used. stomata - are pores under leaves

What is the buoyant force of an object submerged in water according to the law of Archimedes?

Fb = displaced liquid weight According to the law of Archimedes buoyant force of a submerged object is equal to the weight of the displaced liquid.

When a voltaic cell is constructed, one electrode consists of a silver strip in a solution of AgNO3, and the other has an iron strip placed in a solution of FeCl2. The overall cell reaction is :

Fe(s) + 2 Ag+1 (aq) à Fe3+(aq) + 2 Ag (s) What is being oxidized, and what is being reduced?

Fe is being oxidized and Ag is being reduced

Fe(s) is oxidized to 3+, and Ag is reduced to 0. Oxidation is to lose electrons and Reduction is to gain electrons. A voltaic cell produces an electric current if part of a complete circuit

One strucutre, the antheridium, produces male nuclei. The other structure, the oogonium, produces female nuclei.

Fertilization, or sexual fusion, occurs within the oogonium, and the spores that form develop into new organisms.

Which of the following is NOT a behavior that reduces violent encounters in populations?

Fighting Dominance hierarchies can prevent combat by establishing a "pecking order" to determine who gets first access to resources.

Which of the following is NOT a behavior that reduces violent encounters in populations? Rituals. Territoriality. Dominance hierarchies. Fighting.

Fighting Dominance hierarchies can prevent combat by establishing a "pecking order" to determine who gets first access to resources. Territoriality can reduce violent encounters by establishing which animals have claim to the resources in a given area. Ritual behaviors can be used to reduce violent encounters by competing for resources using symbolic tests of strength, and displays.

What is the horizontal force exerted on a 700 kg race car if it is driven at 100 km/hr on a curve with a radius of 600 m?

First convert the speed from km/hr to m/s: 100 km/hr x 1000 m/km x 1 hr/3600 sec = 27.8 m/s Force = mass x centripetal acceleration = m v2/R = 700 kg x (27.8 m/s)2 / 600 m = 900 N

Tail-like projections formed on the outside of a motile cell's membrane are known as which of the following?

Flagella Cilia and flagella are slender projections which are specialized to serve a variety of functions. Flagella are tail-like projections which protrude from the cell bodies of certain prokaryotic and eukaryotic cells and function in locomotion. Prokaryotic and eukaryotic flagella differ in protein composition, structure, and mechanism of propulsion.

Who discovered that a bread mold known as penicillin could inhibit bacteria growth? Pasteur. Fleming. Jenner. Spalanzani.

Fleming. Explanation: Alexander Fleming accidentally discovered a mold colony that had contaminated his bacteria plate. The mold was inhibiting the growth of the bacteria, and said discovery led to modern-day antibiotics.

Which of the following is the most electronegative element?

Fluorine has an electronegativity value of 4.0, which is the highest on the Periodic Table.

This figure summarizes Lenz's law. What will be produced by the induced current?

Flux According to Lenz's law, magnetic flux will be produced by the induced current.

A geo-stationary satellite orbits the Earth in a circular orbit of 36000 km. How long will the time period be of a spy-satellite orbiting a few hundred kilometers above the Earth's surface? (Earth's radius is 6400 km).

For a satellite of mass, m moving with a velocity, v in a circular orbit if the radius, r around the Earth of mass, M, we have mv2/r=G•m•M/r2, or v=√(GM/r). Now, v=2πr/T. Thus, 2πr/T=√(GM/r), or T=kr(3/2). Therefore, T2/T1=(r2/r1)(3/2). In the question r2=6400km and r1=36000km. For a geostationary satellite, T1=24hrs. Using these T2=24x(64/360)(3/2)≈1.8 hours. The closest option is C.

When would the magnifying power of a telescope be high? The objective has a long focal length, and the eye-piece has a short focal length.

For a telescope, the magnification is M=fo/fe. To maximize magnification therefore, the objective should have a long focal length, and the eye-piece should have a short focal length.

An air mass is a large volume of air that develops the characteristics of the area where it forms. These characteristics can include humidity and temperature.

For instance, if an air mass forms in Siberia, then the air mass will develop cold temperatures and have dry humidity in the winter.

A projectile can be fired a maximum range of 200 m. What is the maximum height attained by the projectile?

For maximum range the angle of launch should be 45°. Hence Rmax=v02/g and hmax=v02/4g. Rmax= 200 m. hmax = Rmax/4 = 200/4 = 50 m. Therefore, the correct option is b.

It must provide information on a large amount of data. It must be repeatable, and have comparable information.

For that reason, the best experiment to conduct is probably to have five students study for three hours and five students not study, and then record all of the students' results.

If the mass of a particle is 9.1 x 10-28 kg and it is fully converted into energy, then what will be the final quantity of energy? 8.19 x 10-11 J

From energy equation E= mc2 we get, = 9.1 x 10-28 kg x (3 x 108 m/s)2 =81.9 x 10-28 x 1016 = 81.9 x 10-12 J = 8.19 x 10-11 J

Atomic number is an incorrect choice as this represents the number of protons that are located in an atom's nucleus.

Further isotopes have atoms that are made of the same elements but with different mass numbers. Finally, ion is an incorrect answer choice since an ion describes an atom that gains or loses an electron.

Cleavage is an incorrect choice as minerals that have a cleavage breaks and have uneven planes.

Further, fluorescence is an incorrect choice as fluorescence happens when a mineral interacts with an ultraviolet light and the light causes to mineral to glow in the dark

The global winds carry colder air to warm areas located near the equator. Also, global winds carry warmer air to colder regions near the poles.

Further, global wind system assist with stabilizing thermal energy on the Earth.

Outwash is an incorrect choice as this is an area that is at the edge of a glacier and the outwashed sediments are deposited into streams.

Further, kames are incorrect since kames are a conical mound of sediment that is layered and deposited by streams. Then, eskers is an incorrect choice as this is a long, winding ridge of sediment that is layered and deposited by streams.

Convection is the motion of matter including thermal energy that results from the changes in temperature. Ridge push is an incorrect choice as this is a result of the weight of the lifted ridge pushing the oceanic plate to the trench that is created at the subduction zone.

Further, slab pull is an incorrect choice as this is the weight of the subducting plates that pulls the slab trailing it into the subduction zone. Then, hot spot is an incorrect choice as a hot spot is the heat in the mantle that has the ability to produce magma, which rises to Earth's surface. v

Basaltic magma has 42-52% of silica in it. And this type of magma is located in the Hawaiian islands.

Further, the amount of silica in magma impacts the temperature and how fast magma flows.

A term used in popular culture to describe the psychological reaction of an individual or society to the fast pace of technological or social change is called which of the following?

Future shock Future shock is a term that was coined by writer Alvin Toffler in 1970. It describes the psychological reaction of individuals or a society, to the fast pace of technological and social change.

Which person in history can be considered to be the Father of the Modern Scientific Method?

Galileo

This type of epithelium is specialized for absorption or secretion. Glandular. Mucous membrane. Stratified. Basement membrane.

Glandular Epithelial tissue line the inner and outer surfaces of the body. Epithelia may be specialized to certain function. Glandular epithelia are specialized for absorption or secretion.

Sir Isaac Newton discovered something called the force of gravity. In which example provided would you expect the gravitational attraction force to be the greatest?

Gravitational attraction is related to mass and distance, so the attraction between the Earth and the moon would be the greatest. The tidal forces of the moon that drive all of Earth's tides prove that.

Which of the following statements does not support the notion that plants evolved from green algae?

Green algae does not have jacketed reproductive organs. In fact, the development of jacketed reproductive organs is believed to be one of the major evolutionary developments of the plant.

A gamete has how many times the number of chromosomes as a somatic cell?

Half Meiosis is a variation of cell division that occurs to produce a sex cell, or gamete. During meiosis, a daughter reproductive cell or gamete (egg, sperm) is produced that has half the number of chromosomes as the parent cell.

One of the chief functions of hemoglobin in the blood is to do which of the following?

Hemoglobin in the red blood cells has a strong affinity to attach to oxygen molecules. The blood circulating past the lungs collects oxygen and carries it to all the cells of the body.

In taxonomy, what is the difference between homology and analogy?

Homology is likeness based on shared ancestry and analogy is similarity due to similar adaptation Homology is likeness based on shared ancestry. In general, the more homology between two species, the more closely they are related. Analogy is similarity due to convergent evolution. In analogy, similarity arises when two unrelated species develop similar features due to similar ecological roles that have led to natural selection selecting for similar adaptations to the environment.

Which of the following best defines the Rivet Model of Communities? A rivet community is one in which there are close relationships between different species.

However, modern ecologists generally do not believe in that model.

The Coriolis Effect causes what physical phenomena? Hurricanes turn clockwise in the Southern Hemisphere. Bath tubs drain clockwise in the Northern Hemisphere. The jet stream turns clockwise in the Northern Hemisphere. Water current in the Southern Hemisphere spins counterclockwise.

Hurricanes turn clockwise in the Southern Hemisphere. The Coriolis Effect is a natural angular momentum applied to large-scale objects and is produced by the rotation of the Earth. The effect causes a clockwise spin for air and water masses in the Southern Hemisphere, and a counterclockwise spin for large objects in the Northern hemisphere.

What is the difference between a saturated fatty acid and unsaturated fatty acid? Carbon atom Oxygen atom Hydrogen atom(s) Nitrogen atom

Hydrogen atom(s) Explanation: Saturated fatty acids have all single bonds and are filled with as many hydrogen atoms as possible. Unsaturated fatty acids have a double bond which means it is missing one or more hydrogen atoms.

Which of the following are the molecular equivalent of Velcro? Correct answer: hydrogen bonds

Hydrogen bonds got its name because the hydrogen atoms on polar molecules are often seen sticking to atoms on other polar molecules, like Velcro.

Which of the following is NOT a parasitic plant? Mistletoe. Haustoria. Hyphae. Dodder.

Hyphae Parasitic plants invade a host plant and siphon xylem or phloem sap from its vascular tissue. Example of parasitic plants are: mistletoe, dodder and haustoria.

Which of these elements and compounds form +1 cations in ionic bonding? I. Hydrogen II. Any alkali III. Nitrogen IV. Copper

I, II, III, and IV. Alkalis are the elements in the first column of the Periodic Table, and all have one electron in their outer shell, giving them a charge of positive one. Hydrogen also is in the first column, even though it is not an alkali. Nitrogen has a charge of negative 3, so when it combines with four plus one hydrogen ions, the molecule is given a charge of plus one. Finally, copper is one of the transition metals that has a charge of positive one. There is no pattern relating to the charges of these elements; they just need to be learned.

Which systems can benefit from technological improvement? I Production storage and disposal of consumer products; II Transportation; III Power generation and energy consumption. I only. I and II only II and III only. I, II, and III.

I, II, and III Technology can be applied to all three to improve them. Any improvement in the management of energy or resources is helpful.

The indicator should be chosen so that the end point is approximately the same as the equivalence point (at which equivalents of acid = equivalents of base), but the indicator used has no effect on the equivalence point.

If a weak acid and/or weak base is used, then the equivalence point pH will not be 7.0. It would take less of a higher concentration titrant to reach the end point, not more.

A radioactive substance has a half-life of two (2) years. Having originally 48 moles, how many moles would remain after eight (8) years?

If the half life of a substance is two (2) years, that means that the mass will halve every two (2) years. Then after two (2) years, the mass will have halved to 24 moles. After another two (2) years, it will become 12 moles; then six (6) moles; and finally three (3) moles will remain after eight (8) years.

Levers are a simple tool that can give a mechanical advantage. What force is required to lift the weight at the arrow?

If the lever is fulcrumed at the halfway point, there will not be a mechanical advantage, and the force will be equal to W to move it. If the lever is fulcrumed closer to the weight, there will be a mechanical advantage.

A mutation is a change in the genetic information of a cell or virus. It may involve a large portion of the chromosome, or it may affect just one or a few nucleotides.

If the mutation occurs in the gamete, or sex cell, it may be passed on to offspring.

What will happen if a bowling ball and a golf ball are dropped from a high bridge?

If you remember Galileo's experiment from the Leaning Tower of Pisa, the mass or size of an object does not affect its acceleration. The balls should fall at roughly the same speed with slight differences due to air resistance.

Which of the following persons proposed that new cells arise only from previously existing cells, thus disputing the idea of spontaneous generation? Correct answer: Virchow

In 1858 Rudolf Virchow proposed that new cells arise only from previously existing cells,

Identify the statement that correctly describes what is happening when a pebble is dropped in a still pool and a wave motion is formed which travels away from the point of the drop of the pebble. The water particles do not travel in the direction of the wave motion

In a wave motion, it is the disturbance which travels forward and not the medium. The particles of the medium go up and down about their mean positions, which mean it is not stationary either, and each particle hands over a part of its energy to the succeeding particle.

Between cells in most animal tissues, gap junctions are formed by which of the following proteins? Correct answer: connexins

In almost all animal tissues, gap junctions are formed by proteins called connexins, that form little rings in the membranes of cells. The rings on two adjacent cells line up with each other and

In fluid dynamics, what is the differential form of the continuity equation? Assume ρ is fluid density, t is time, and u is fluid velocity.

In fluid dynamics, the differential form of the continuity equation is ∂ρ/∂t+ ∇•(ρu)=0, in which ρ is fluid density, t is time, and u is fluid velocity.

Which of the following do fermentation and cellular respiration have in common? Glycolysis Glycolysis is the first step in the process of cellular respiration.

In glycolysis, glucose is broken into two molecules of pyruvate. Glycolysis also occurs as the first process in fermentation. In glycolysis, the oxidizing agent is NAD+, not oxygen, .

Many mammals testes are held outside the abdominal cavity in the scrotum to keep the temperature about two degrees Celsius below the body temperature.

In many rodents the testes are drawn back into the abdominal cavity to interrupt sperm maturation between breeding seasons. Some mammals whose body temperature is low enough to allow sperm maturation, retain the testes within the abdominal cavity.

A telescope has an objective of focal length of 100 cm, and an eye-piece of focal length of 5 cm. What is the magnifying power of the telescope when both the object and the image are at infinity?

In normal adjustment, (i.e. when both the object and the image are at infinity), the magnification M=fo/fe=100/5=20.

During photosynthesis, the plant uses visible light energy to fix carbon and manufacture simple sugars and grow. The carbon to build these simple sugars comes from which of the following?

In photosythesis. the plant builds simple sugars by fixing carbon from carbon dioxide in the air. The carbon in carbon dioxide is pulled off and connected to a sugar molecule, leaving oxygen.

The Golgi apparatus, or the Golgi body, is defined as

Incorrect a structure that assists with cell division the membranous network in the endoplasmic reticulum the membranous network that synthesizes lipids and carbohydrates

A constant voltage is applied across a wire which produces some heat in the wire. If a second wire of the same material but twice as long as the first is used, the heat produced in the second wire will remain the same as in the first wire. What must the area of the cross-section of the second wire be?

Increased by a factor of two (2) Heat produced=V2/R=V2A/ρl, where ρ is the specific resistance of the wire, which is the same for both the wires. The heat produced therefore, will be the same in the second wire of length 2l, if its area of cross-section A is doubled.

What scientific process uses deduction to explain what has been observed?

Inference is the process of coming up with or deducing an explanation for something based on observations which have been made. Prediction relates to anticipating the outcome of a process rather than describing its cause or mechanism.

An open circulatory system would be found in what? Annelid. Mammal. Bird. Insect.

Insect An open circulatory system would be found in insects, other arthropods and most mollusks. In an open circulatory system, hemolymph is in spaces between organs and bathes the internal tissues which allows for chemical exchange. Most annelids, some mollusks, and vertebrates have a closed circulatory system.

In what category does Zero Error belong?

Instrumental errors Place a blank sample in the instrument for which we know it should 'read zero'. When the instrument reads a number different than zero, that number is known as Zero Error.

Which of the following integrate sensory information and send out responding signals? Correct answer: interneurons

Interneurons within the CNS integrate sensory information and send out responding signals. When an interneuron receives an impulse from a sensory neuron, the interneuron determines what response to generate. If a response is required, the interneuron passes the impulse on to motor neurons.

Which of the following depicts the amphibian circulatory system? Image C This depicts the amphibian circulatory system. Amphibians have a 3-chambered heart that consists of two atria and one ventricle. Blood from the lungs is called pulmonary flow.

It goes to the left atrium. Blood from the body is called systemic flow. It flows into the right atrium. Blood flows from both atria into the ventricle. This system provides the organism with a gas exchange

What does the name "streptococcus" indicate about the shape of this bacteria? It is spiral in shape. It is an elongated shape. It has multiple spheres in a chain. It has a flagellum.

It has multiple spheres in a chain. Explanation: Correct answer: It has multiple spheres in a chain Bacteria are generally categorized in three shapes: Cocci are spherical, bacci are rod shaped and spirilla are spiral in shape. Bacteria may be found in clusters and are named based on the arrangement of the cells in the cluster. For example: Staphylococcus - staphylo = a bunch of grapes, coccus = berries/spheres. Streptococcus - strepto = chain, twisted, coccus = berries/spheres.

Why was fixing rather than scrapping the Hubble Space-Based Telescope a good political decision? It would become space debris if not used. It gives the astronauts something to do. It is less expensive to fix than to replace. It has provided amazing pictures of the universe.

It is less expensive to fix than to replace. Explanation: All things considered, the Hubble Space-Based Telescope has been an amazing asset to astronomy. Building another one like it would be extremely expensive. At this point, it is less expensive to fix it than to replace it.

Identify a purpose of a potentiometer.

It is used as a variable resistance in circuits A potentiometer is a three-terminal resistor with a sliding contact that forms an adjustable voltage divider. If only two of the three terminals are used, it acts as a variable resistor.

Why should the use of platinum wire be avoided to test the lead salts?

It reacts with lead Because lead combines with platinum and the wire gets corroded.

How does the speed of sound change in water? It speeds up. It slows down. It stays the same. It sometimes goes faster and sometimes goes slower.

It speeds up. Unlike light, sound travels faster in water because of the density of particles. A scuba diver cannot discriminate the direction of sound under water because it hits both ears at almost the same time.

What type of behavior will be observed in the hydrophilic head of a phospholipid?

It will stay in contact with the water Phospholipids have hydrophilic heads and hydrophobic tails. When phospholipids come into contact with water, they will assemble into aggregates. The hydrophilic heads will be on the outside in contact with the water and the hydrophobic tails will restrict themselves to the inside in the water-free middle.

What type of behavior will be observed in the hydrophilic head of a phospholipid? It will stay in contact with the water. It will remain remote from the water. It will form a hydrogen bond with water. It bonds with oxygen.

It will stay in contact with the water Phospholipids have hydrophilic heads and hydrophobic tails. When phospholipids come into contact with water, they will assemble into aggregates. The hydrophilic heads will be on the outside in contact with the water and the hydrophobic tails will restrict themselves to the inside in the water-free middle

Every day an average of two people are born for every one that dies, leaving a ratio of 2 to 1 for births to deaths. What would happen to this ratio if the average life expectancy were to double?

It would go to 4 to 1. Doubling life expectancy would lower the death rate by about one-half. This would change the ratio to two (2) to 0.5, or four (4) births to one (1) death.

Biofeedback is a treatment technique in which people are trained to improve their health by using signals from their own bodies. People learn, for example, to read devices that "feed back" information about their body's condition. Physical therapists use biofeedback to help stroke victims regain movement in paralyzed muscles. Psychologists use it to help tense and anxious clients learn to relax. Specialists in many different fields use biofeedback to help their patients cope with pain.

Jumping rope Jumping rope, in itself, is not a biofeedback technique. It does not measure signals from the body.

Biofeedback is a treatment technique in which people are trained to improve their health by using signals from their own bodies. People learn, for example, to read devices that "feed back" information about their body's condition. Physical therapists use biofeedback to help stroke victims regain movement in paralyzed muscles. Psychologists use it to help tense and anxious clients learn to relax. Specialists in many different fields use biofeedback to help their patients cope with pain. Which of the following is NOT an example of the use of biofeedback? Measuring electrical impulses Stepping on a scale Jumping rope Taking your temperature

Jumping rope Jumping rope, in itself, is not a biofeedback technique. It does not measure signals from the body

Which of the following organs removes cell waste from the blood? Heart Large Intestine Small Intestine Kidney

Kidney Explanation: The correct answer is (D). The kidney is the organ that removes cell waste from the blood. The heart is responsible for pumping blood throughout the body. The small intestine is responsible for the majority of digestion, absorption of nutrients, and peristalsis. The large intestine is the final stage of digestion and it absorbs remaining water as well as rids the body of solid waste.

The oldest clam just recently found in the Arctic Ocean was 405 years old. Why is it important to study that clam ?

Knowing how it got so old might help us learn about living longer. Learning how a clam grew to the ripe old age of 405 might help man extend his average life span. That is a very practical application of science to society

How does radioactive dating in sediments work?

Knowing the half-life of a radioactive substance and the ratio of its initial isotopes, the elements present in it today can be measured and calculated in relation to how long it has been decaying. Different elements are used for different time scale measurements.

Why would the discovery of fossil evidence that modern humans arrived in the Americas around 36,000 years ago be important to our modern society ? It might provide the missing evolutionary link. This evidence might provide clues about the dinosaur era. Knowledge about human history can teach us more about the current situation, and how it came about. Fossil evidence about man's origins can tell us how the universe was created.

Knowledge about human history can teach us more about the current situation, and how it came about. Humans have always been curious about where they came from, and better knowledge of human history can help us to better understand how things are today. The other answers are incorrect: according to current scientific estimates from the fossil record, all of the other events described in the answers occured far more than 36,000 years ago.

Which of the following terms or quotations is consistent with Lamarck's Theory of Evolution?

Lamarck believed that animals could change their heritable material through their actions. He felt that since giraffes stretched to reach high branches of trees, over time their offspring would have longer necks. That same Theory would suggest that bodybuilders, by increasing muscle mass, would subsequently impart strength to their children. That view is no longer accepted by science. The correct answer choice therefore, is (B).

A galvanometer of resistance 10Ω gives full scale deflection when a 1mA current passes through it. What is the resistance required to convert it into a voltmeter reading of up to 2.5 V?

Let R be the resistance to be connected in series with the galvanometer of resistance G in order to convert it into a voltmeter reading of up to 2.5V. The current through the galvanometer then is Ig=2.5/(R+G). Putting Ig=1mA=10-3A and G=10Ω, we get R=2490Ω.

What is the term given for the electron dot representation of covalent molecules?

Lewis structure A Lewis structure can be drawn for any covalently bonded molecule, as well as coordination compounds.

What type of spectrum do X- rays have?

Line spectrum. Wavelengths of X-rays are determined by transitions of anticathode atoms from higher to lower energy levels. Since the properties of X-rays depend on atoms of anticathode, X-rays are named characteristic and their spectrum is discrete (line).

Which term may be used to describe the pH scale? Logarithmic. Additive. Linear. Proportional.

Logarithmic. Explanation: The pH scale is logarithmic. That means that for each one step down the scale, the element or compound is ten (10) times more acidic. For example, a solution that has pH 4 is ten (10) times more acidic than a solution with pH 5.

Geothermal energy is made possible by the existence of which of the following? Wind Oil Coal Magma

Magma Although wind, oil, and coal are all sources of energy, it is the magma located 2 to 12 miles beneath the Earth's surface that is the source of geothermal energy.

Many geological minerals are formed by which of the following processes?

Many minerals are formed in the crystallization process, in which hot liquid solutions rich in minerals cool to form mineral crystals.

Which of the following features of a map represents the distances on a map compared to the distances on the surface of the Earth? map scale A map scale is the ratio between the distances on a map and the distances on the surface of the Earth.

Map scales are measured in SI and sometimes measured in miles and inches. A map legend is an incorrect choice as this explains the types of symbols on a map. Further, map graphic scales is an incorrect choice as these are lines that show a specific distance such as 6 miles.

The first relationship between electricity and magnetism was established by which of the following?

Maxwell's (four) equations established the first quantitative relationship between electrical forces and magnetic forces. They also show that light is an electromagnetic wave.

Which RNA molecule carries genetic information from DNA to protein molecules?

Messenger RNA Messenger RNA (mRNA) is the RNA that carries information from DNA to the ribosome, the sites of protein synthesis (translation) in the cell.

If you placed an animal in a calorimeter and measured heat loss, you would be measuring its what?

Metabolic rate The total energy an animal uses in a given amount of time is its metabolic rate. Energy is measured in calories or kilocalories. Metabolic rate can be measured by placing an animal in a calorimeter and measuring its heat loss.

If you placed an animal in a calorimeter and measured heat loss, you would be measuring its what?

Metabolic rate The total energy an animal uses in a given amount of time is its metabolic rate. Energy is measured in calories or kilocalories. Metabolic rate can be measured by placing an animal in a calorimeter and measuring its heat loss.

If you placed an animal in a calorimeter and measured heat loss, you would be measuring its what? Temperature. Body Mass Index. Metabolic rate. Body Heat Index.

Metabolic rate The total energy an animal uses in a given amount of time is its metabolic rate. Energy is measured in calories or kilocalories. Metabolic rate can be measured by placing an animal in a calorimeter and measuring its heat loss.

Which of the following is NOT a function of smooth endoplasmic reticulum (ER)?

Metabolism of proteins Smooth ER has various functions in metabolic processes. These functions include: synthesis of lipids, metabolism of carbohydrates, and detoxification of drugs and poisons. Enzymes in the ER are critical to the synthesis of lipids.

Buildings inside a city have the ability to create a microclimate Explanation: The correct answer is microclimate.

Microclimates can develop in cities because of buildings. With microclimates, it becomes cooler with increased elevation. So in a city with buildings, the climate can become cooler due to the elevations of buildings and in some cases concrete in a city.

Identify the name given to the experiment that showed it is possible for organic compounds to be synthesized from inorganic reagents in conditions similar to what were present on primitive Earth.

Miller-Urey experiment

Mica is a dominant mineral for making furnace windows. What mineral group does Mica belong?

Minerals are grouped by their chemical properties. The major mineral groups include silicates, sulfides, oxides, sulfates, halides, carbonates and native elements. The mineral mica belongs to the silicate group.

A mineral that breaks and exhibits a jagged edge is known to have a

Minerals that break unevenly across a rough or jagged edge is called a fracture. .

In eukaryotic cells, the enzymes for cellular respiration reside within the what?

Mitochondria In eukaryotic cells, the enzymes for cellular respiration reside within the mitochondria. This keeps the enzyme molecules localized and concentrated, which makes cellular respiration more efficient.

Mosses and their relatives are generally called which of the following? Correct answer: bryophytes

Mosses and their relatives are generally called bryophytes, or nonvascular plants. Unlike all other plants, these organisms do not have vascular tissues, or specialized tissues that conduct water and nutrients.

Which of the following statements regarding the hydrologic cycle is incorrect?

Most of the water evaporated from oceans is returned by rainfall. While some water is returned by runoff, the overall net movement of water is from land to oceans. The correct answer choice therefore, is (C).

Metabolic rates can change based on age, sex, size, activity level, time of day, among other factors

Most terrestrial animals have a metabolic rate that is 2 to 4 times their body mass index.

The process through which favorable traits become more common in a species and unfavorable traits become less common is called

Natural selection Natural selection is the gradual process by which biological traits become either more or less common in a population.

The interaction of an organism's genetic make-up and its environment is called which of the following?

Nature versus nurture. Nature is the genetic make-up of an organism, and nurture is its environment. Those two components determine how an organism develops.

Which molecule would tend to migrate towards the positively charged electrode in a pH 7 solution?

Negatively charged ions move towards positive electrodes and vice versa. Potassium, sodium and lysine are all positively charged and will not move towards the positive electrode. DNA has a phosphate backbone which gives the molecule a negative charge. That is why DNA electrophoresis gels are set up so that the wells are near the negative charge so that the DNA samples can migrate towards the positive electrodes.

Which of the following is unique to animals?

Nervous tissue Animals are multicellular organisms. Most use ingestion as their means of nutrition. Structural proteins such as collagen bind animal cells together. Muscle and nervous tissues are unique to animals.

Which of the following is unique to animals? Cells. Nervous tissue. Ovaries. Sexual reproduction.

Nervous tissue Animals are multicellular organisms. Most use ingestion as their means of nutrition. Structural proteins such as collagen bind animal cells together. Muscle and nervous tissues are unique to animals.

The following is an excerpt from the U.S. Declaration of Independence: "When in the course of human events, it becomes necessary for one people to dissolve the political bands which have connected them with another, and to assume among the Powers of the earth, the separate and equal station to which the laws of Nature and of Nature's God entitle them, a decent respect to the opinion of mankind requires that they should declare the causes which impel them to the separation." Which famous person's ideas inspired the reference to the "laws of Nature" mentioned above? Plato Aristotle Newton Galileo

Newton Explanation: The correct answer is (C), Newton. Sir Isaac Newton (1642-1726) was a prolific scientist whose thoughts were respected in the colonies. His Principia Mathematica (1687) described the relationship of science with nature.

Another name for the manner in which an organism makes a living is called which of the following? Ecology. Habitat. Ecosystem. Niche.

Niche A niche is a term that refers to the relational position of a species to its ecosystem, and that can also be referred to as the manner in which an organism makes a living.

The earth's atmosphere consists of three major gases . Which is the most abundant gas? Carbon dioxide. Oxygen. Nitrogen. Argon.

Nitrogen. Explanation: Approximately 78 % of the earth's atmosphere is composed of nitrogen.

As the ozone layer gets thinner day by day, is it safe to produce it on ground level and let it escape to recover it?

No, never Ozone in the upper atmosphere shields us from ultraviolet radiation. However on ground level, it is highly toxic to both plants and animals as it can damage lungs. It can bring on coughing; asthma attacks and lowers the immune system.

Consider the following gas reaction: H2(g) + Br2(g) ßà 2 HBr (g) The concentrations of H2, Br2, and HBr are 0.05 M, 0.03 M, and 500.0 M respectively. The equilibrium concentration constant for this reaction at 400oC is 2.5 X 103 . Is this system at equilibrium?

No, the reaction must shift to the left to reach equilibrium

Which two ends in a magnet will attract each other?

North to south Unlike poles attract, and like poles repel in a magnet. That means that the north and south pole of a magnet attract.

Nuclear reactors are capable of producing large amounts of energy. What process produces it? Radioactive decay. The burning of coal. Nuclear fusion. Nuclear fission.

Nuclear fission. Explanation: Just like an atom bomb, nuclear power plants produce energy from nuclear fission. Fission is the splitting of large atoms to produce smaller atoms and energy. If a chain reaction were to occur in the fission process, it is slowed down and controlled in a nuclear reactor, and out of control in an atomic bomb.

Very large amounts of energy are produced in atomic bombs. What is the process that powers an atomic bomb called? Nuclear fission. Nuclear fusion. Radioactive decay. Chemical reaction.

Nuclear fission. Explanation: The splitting of the atom is what powers an atomic bomb, and the process is called nuclear fission.

Cell membranes can block the path of hydrophilic things like ions and charged molecules, so they need to go through channels, similar to the doors in the theater. the ions just flow through by themselves with no extra help in the form of pumping action.

Once the channel (or door) is open, This channel-opening process is called facilitated diffusion because ions and charged molecules need to be helped across the membrane by channels. Since ions and charged molecules need to go through channels, their rate of diffusion can only get so high.

How much louder is a 120-decibel sound than a sound than a 100- decibel sound? One hundred (100) times louder.

One hundred (100) on the decibel scale is 1010, and 120 is 1012. So a sound that is 120 is 102, or 100 times louder than a sound that is 1010. The decibel scale increments a power of ten for every ten (10) decibels, so 160 would be 1016, and 40 would be 104.

What is the molar volume of any gas at STP?

One mole of any gas, oxygen, hydrogen, carbon dioxide, helium, nitrogen, at Standard Temperature (0oC) and Standard Pressure (1.0 atm) occupies 22.4 Liters. At room temperature (20oC) the molar volume is 24.4. Liters.

Which of the following species mentioned below has maximum number of chromosomes?

Ophioglosum Ophioglosum is a pteridophyte and it posses the maximum number of chromosome 2n=1656 .On the other hand Haplopappas has the lowest number of chromosome 2n=4.Felis catus and Triticum aestivum posses 42 and 38 respectively.

Which of the following is NOT a difference between humans and other hominids?

Opposable thumb. Explanation: Correct answer: Opposable thumb The anthropoid primates have an opposable thumb. This thumb functions as a power grip in apes and monkeys, but is suited for more precise manipulation in humans. Humans are different from other hominids in that they have a great duration of parental care, long term pair bonding and an exceptional dependence in their newborn infants.

A researcher decides to experiment with different levels of light to ascertain whether light starvation impacts the quality of chlorophyll produced in plant leaves. He decides to change the light levels three different times; exposing some plants to no light, some receiving full light from a lamp, and some receiving the light through a piece of material to dim the light. Which scale would be used to measure the amount of light received by each plant?

Ordinal Explanation: There is no numerical value placed on the measurement of light, so it is not a quantitative scale (continuous or discrete). There is however an order to the values i.e. they could be placed on a scale from light to dark, which means that the values are not nominal but in fact ordinal.

Osmosis is a form of which of the following?

Osmosis is another name for diffusion with water. The process falls under a group of cellular transport processes called passive transport.

What will isotonic solutions have in common at a temperature of 40°C?

Osmotic pressure Isotonic solutions have same osmotic pressure. For two solutions: Π1 = c1RT ; Π2 = c2RT Therefore, Π1 = Π2 and c1= c2

Identify a base according to the Lewis theory.

PCl3 According to Lewis theory, bases are species that can donate electron pair. In this case only the phosphorus atom is an electron pair donor.

Which of the following molecules is the true end product of photosynthesis and is capable of forming glucose, lipids, and proteins?

PGAL PGAL (or phosphoglyceraldehyde) can be used as a starting point for the production of glucose, RDP, proteins, and lipids. Hydrogen from the light reactions combines with PGA to form six new 3-carbon compounds which are called PGAL. Three carbon dioxide molecules enter at the start of the cycle to be followed by an additional three molecules of carbon dioxide. Each turn around the cycle can spin off a PGAL to eventually form glucose.

What is the hydroxide ion concentration ( [OH-] ) in a solution with pH = 4?

Perhaps the easiest way to solve this problem is to use the following equations: pH + pOH =14, and [OH-] = 10-pOH. Thus, pOH =14 - 4 = 10 and [OH-] =10-10.

Which period of elements has four (4) gases?

Period Two (2) has four gases: Nitrogen, oxygen, fluorine, and neon. Period Three (3) has two (2) gases, and Period Four (4) has one gas.

The structures labeled "W" are what? Bases. Nitrates. Sugars. Phosphates.

Phosphates The phosphate anion is made up of one phosphorous and four oxygens. The molecular formula for the phosphate anion is O4P3-.

Which statement is false? Placeholders are significant. All non-zero digits are considered significant. Leading zeros are not significant. Additional zeros to the right of the decimal are significant.

Placeholders are significant Placeholders are not significant. E.g.: For number 0.0003, number of significant figure is 1.

A localized group of individuals of the same species is called what?

Population A population is a localized group of individuals of the same species. Although natural selection acts upon the individual, it is ultimately populations that evolve.

A total of 90 percent of the geologic time that encompasses three of the earliest eons is called? Jurassic Triassic Precambrian cretaceous

Precambrian The correct answer is Precambrian. The Precambrian time is the earliest eon and is comprised of the Hadean, Archean and Proterzoic times. Also, the Precambrian time makes up 90 percent of the geological time.

Accuracy is the degree of closeness of a measured quantity to its actual (true) value.

Precision is the reproducibility or repeatability to which further measurements or calculations show the same or similar results. Both are important to scientific methodology.

Which factor does not contribute to the difference in the speed of sound?

Pressure Except pressure, all other factors greatly contribute to difference in sound speed.

Which of the following will most likely be the result if a protein denatures? Correct answer: the protein will no longer function

Protein shape is essential for the protein's ability to do its job and each one is carefully folded, creating unique sections that are tailored for their particular job. If a protein denatures, or unfolds, it will no longer function.

Which of the following is NOT a function of a plant cell vacuole?

Pump water out of the plant cell The vacuole in a plant cell has a variety of functions. It can be used to store organic compounds, and inorganic ions. It can be a disposal site for metabolic by-products. It can contain the pigments that give flowers their color, or can contain poisons that protect the plant against predators. It can absorb water which helps in plant growth by elongating the cell with minimal investment in new cytoplasm.

If you put more energy into a wave, what happens to its wavelength and frequency?

Putting energy into the wave causes the frequency to go up, and the wavelength to go down. High frequency waves are more energetic.

What causes the color of the sea to appear to be blue?

Rayleigh scattering of light The sea appears blue because the water droplets of the sea act as a mirror and reflects the color of the sky. If the sky is cloudless or if it is a clear sunny day, then water reflects blue color of the sky. This makes the sea water blue.

An allosteric site is a specific receptor site on a part of the enzyme molecule that is remote from the active site

Regulatory molecules bind to the allosteric site and change the enzyme's shape and function. This acts as a regulatory process in enzyme activity in the cell.

What can be used to find the direction of magnetic fields produced when current flows through a wire? Fleming's Left-Hand Rule Left-hand grip rule Right-hand grip rule Right-hand fingers rule

Right-hand grip rule Current-carrying wires generate magnetic field lines that form concentric circles around the wire. The direction of the magnetic field in these loops is determined by the right-hand grip rule. If the conductor is gripped in the right hand with the right thumb pointing in the direction of the current, the fingers point in the direction of the magnetic field.

Territoriality can reduce violent encounters by establishing which animals have claim to the resources in a given area.

Ritual behaviors can be used to reduce violent encounters by competing for resources using symbolic tests of strength, and displays.

What cellular structure is represented by number 5?

Rough Endoplasmic reticulum The endoplasmic reticulum (ER) is a eukaryotic membrane-enclosed organelle that forms an interconnected network of tubules, vesicles, and cisternae within cells. The rough endoplasmic reticula synthesize proteins.

What cellular structure is represented by number 5? Golgi body. Rough Endoplasmic reticulum. Mitochondria. Cytoskeleton.

Rough Endoplasmic reticulum The endoplasmic reticulum (ER) is a eukaryotic membrane-enclosed organelle that forms an interconnected network of tubules, vesicles, and cisternae within cells. The rough endoplasmic reticula synthesize proteins.

In the human body which gland produces the enzyme amylase or ptyalin? Correct Answer: Salivary

Saliva contains an enzyme called salivary amylase (ptyalin). Salivary amylase keeps the pH in the mouth basic. Salivary amylase will digest starch in your food to maltose, which will later get digested to glucose in the Duodenum.

Identify the term that is used to denote a selection of a subset of a population in order to conduct a survey study.

Sampling Sampling is concerned with the selection of a subset of individuals from within a population to estimate characteristics of the whole population.

Tara was trying to design an experiment to figure out the best way to do well on a math test. Which experiment might help her learn the most about how to do well?

Scientific methodology can be a very powerful way to learn things, but the experiment must be designed properly.

Tara was trying to design an experiment to figure out the best way to do well on a math test. Which experiment might help her learn the most about how to do well?

Scientific methodology can be a very powerful way to learn things, but the experiment must be designed properly. It must provide information on a large amount of data. It must be repeatable, and have comparable information. For that reason, the best experiment to conduct is probably to have five students study for three hours and five students not study, and then record all of the students' results.

The pH of distilled water should be a value of which of the following?

Seven (7) is a neutral pH value. The pH scale goes from one (1) to 14. A value of one (1) is extremely acidic, and a value of 14 is extremely basic.

During sexual reproduction of water molds, where does fertilization take place? Correct answer: oogonium

Sexual reproduction of water molds takes place in specialized structures that are formed by the hyphae.

First, calculate the mass which she managed to produce in the first experiment, which was 135 x 0.59 = 79.65g.

She then manages to improve that by 24g, which is 79.65 + 24 = 103.65. To find the yield amount, simply calculate (103.65 / 135) x 100 = 76.7%.

What is the SI unit for electrical conductance?

Siemens (S) is the SI unit for electrical conductance. It is the reciprocal of resistance, and can be expressed as 1/Ω.

What causes the sky to look blue? The sunlit sky appears blue because air scatters short-wavelength light more than longer wavelength light.

Since blue light is at the short wavelength end of the visible spectrum, it is more strongly scattered in the atmosphere than long wavelength red light. The result is that the human eye perceives blue when looking toward parts of the sky other than the sun.

A body goes from point A to point B with a velocity of 40 km/h eastward and returns from point B to point A with a velocity of 60 km/h westward. What is the average velocity of the body during the entire journey?

Since the body goes from A to B and returns from B to A, the total displacement is zero. Hence, the average velocity is 0 km/h. The average speed of the body is 50 km/h, but the question asks for the velocity.

Small balls (n), each of mass (m), impinge elastically each second on a surface with velocity (u). What will be the force experienced by the surface?

Since the collision is elastic, the balls rebound with the same speed (so the change in velocity is 2u). The change in momentum of each ball, therefore, is two (2) mu. The change in momentum per second due to n balls= 2mnu. Force is equal to the rate of change in momentum. The force experienced by the surface therefore, is two (2) mnu.

Imagine an astronaut inside a spaceship far from the solar system. Could he determine that the ship is moving without looking out a porthole? Correct answer: No

Since the spaceship is moving at a constant speed and direction, he will not feel the motion of the ship.

Around the end of the 20th century, the US had an ecological footprint of 8.4 hectares per person, and a carrying capacity of approximately how many hectares per person?

Six A study conducted in 1997 estimated that the US had an ecological footprint of 8.4 hectares per person, and a carrying capacity of approximately 6 hectares per person. This means that the US is already using more resources per person that our country can support. Thus, we are utilizing resources from other areas of the globe, over utilizing the world's resources. This study suggested that the world's human population is already slightly above the earth's carrying capacity.

Marble is created from the metamorphism of quartz rich sandstone.

Slate, schist and gneiss are incorrect choices as these are examples of foliated metamorphic rocks.

Africa, India and Australia were located near which of the following regions approximately 290 million years ago? South Pole North America Asia Pacific Ocean

South Pole Due to the discovery of glacial deposits found in Africa, South America, India and Australia, Wegener concluded that these areas were located close to the South Pole. Also, Wegener theorized that the thick ice cap deposits found in Africa, India, South America and Australia could have been caused by the South Pole's shift in position although he strongly believed that the landmasses such as Africa, India, South America and Australia had drifted apart.

Spiders have the unique adaptation of spinning silk. This silk is spun by organs called spinnerets. The silk is a protein that is in liquid form in special abdominal glands and solidifies into fibers when spun. The silk is used for more than just spinning webs to catch food.

Spiders also use the silk as drop lines for rapid escape from danger, as egg covers, and even as gift wrapping for food that certain male spiders offer females during courtship. Spiders do not have a pupal stage in their development, nor do they form cocoons.

A molecular complex that snips an intron out of the RNA transcript and rejoins the adjoining exons is what?

Spliceosome A spliceosome is a molecular complex composed of small nuclear ribonucleoproteins and small nuclear RNA. Spliceosomes snip an intron out of the RNA transcript and connect adjoining exons.

Identify an irreversible process. Spontaneous flow of energy from a colder body to a hotter body A process or cycle such that the net change at each stage in the combined entropy of the system and its surroundings is zero A process that leads to a decrease in entropy of a system and the surrounding entropy is zero Spontaneous flow of heat energy from a hotter body to a colder body

Spontaneous flow of heat energy from a hotter body to a colder body Explanation: According to the Second Law of thermodynamics,entropy tends to increase. Two lukewarm bodies have a higher entropy than one warm and one cold body. Therefore heat flows from hotter bodies to colder bodies, and this is irreversible as heat energy cannot pass from a cold body to a hotter body spontaneously thus making the cold body colder.

Which of the following is not made up chiefly of amino acids?

Starch is a storage molecule for glucose or other sugars, particularly in plants. The other molecules are comprised almost entirely of protein. Collagen is a structural protein as opposed to a globular protein (e.g. enzymes).

Which of the following statements are true when a change in the chemical structure of a hydrocarbon chain occurs? Statement I: Isomers that have shorter hydrocarbon chains are less volatile. Statement II: Isomers that have a shorter hydrocarbon chain are more volatile. Statement III: Isomers that have a shorter hydrocarbon chain have lower boiling points. Statement IV: Isomers that have a shorter hydrocarbon chain have higher boiling points.

Statements II and III Shortening the hydrocarbon chain as occurs in most structural isomers, makes the molecule more volatile, because it is easier for the molecule to escape a mixture. For example, it is much easier for isobutene to escape liquid than it is for butane to escape, thus giving it a lower boiling temperature due to its higher volatility.

Why is using simple solar distillers a more elegant solution than a more high tech solution for addressing Third World water quality problems?

Sustainable answers to local problems are more likely if they are affordable and can be supported by local technological skills. The solar still is inexpensive, easy to use, does not require electricity, and can easily be supported by the technological skills of a Third World country.

Scientific discipline that studies the classification of organisms based on phylogenetic relationship is:

Taxonomy. Taxonomy is the science of identifying and naming species, and arranging them into a classification. The classification scheme has a hierarchical nature. Hierarchical categories or units are called taxa.

The last step in mitosis is called which of the following?

Telophase The last step in mitosis is called telophase. Cytokinesis, in which one parent cell splits into two identical daughter cells, is a separate process beginning in the final stages of mitosis, but is not part of the mitosis process itself.

Which stage of meiosis decreases a cell's chromosome number from diploid to haploid?

Telophase I A diploid cell has all the homologous pairs of chromosomes. Only after telophase I are there two separate cells, each one containing one member of every homologous pair.

Why are recent breakthroughs in stem cell research so important? Stem cells are remarkable in that they still retain the ability to differentiate into other types of cells.

That is important for brain cell, pancreatic cell, or any other type of cell in the body. In order to make the repairs, raw materials (i.e. stem cells) are necessary.

Alfred Wegener is famous for which geological theory? Plate Tectonics Theory. Uniformitarianism Theory. The Continental Drift Theory. Mineral Hardness Theory.

The Continental Drift Theory Alfred Wegener was the first person to publicly note how the Continents seemed to fit together. This led to the Theory of Continental Drift, an idea not well received at the time.

According to Snell's Law, if the refractive index of two mediums is n1 = 1.5 and n2 = 2.0, what is the speed of light ratio in the two mediums?

The Correct answer is: 4/3 According to Snell's Law, sinθ1/sinθ2=n2/n1=v1/v2. Here, n1=1.5 and n2=2.0. The speed of light ratio therefore, is v1/v2= 2/1.5= 4/3.

a stack of flattened, membranous sacs that chemically processes and packages substances from the endoplasmic reticulum

The Golgi apparatus is a stack of flattened, membranous sacs that chemically processes and packages substances from the endoplasmic reticulum.

The term for the membranous network that receives and transports synthesized proteins (labeled # 5 in the diagram above), is which of the following? Rough endoplasmic reticulum The correct answer is (C). The rough endoplasmic reticulum is the membranous network that receives and transports synthesized proteins.

The Golgi apparatus, also known as the Golgi body, (labeled # 6) chemically processes and packages substances from the endoplasmic reticulum. The smooth endoplasmic reticulum (labeled # 8) is also a membranous network, but it synthesizes lipids and carbohydrates. The lysosomes (labeled # 12) act as the cell's "digestive system."

What stellar diagram plots luminosity against star type?

The Hertzsprung-Russell (H-R) diagram is a plot of relationship between the stars' luminosity versus its spectral type.Those plots tell us alot about the types of stars that exist.

In which of the following organelles does the Krebs cycle occur? Correct answer: mitochondria

The Krebs cycle is the second stage of aerobic cellular respiration, and aerobic cellular respiration takes place in the mitochondria.

Which statement is true? The Laws of Reflection of Light hold true for plane as well as curved reflecting surfaces.

The Laws of Reflection of Light are true for plane, as well as curved reflecting surfaces. All the other options are incorrect.

What is 1 farad equal to? Correct Answer: 1coulomb / 1 volt

The S.I unite of the capacitance is farad And capacitance is C = Q/V 1 farad = 1 coulomb / 1 volt

The modern metric system, known as SI, was invented to solve the problems with the English system. Which of the following is NOT an advantage of the SI system over the English system, the system we use most often in America today?

The SI system has standard units that are based on ordinary, everyday objects that everybody is familiar with.This statement is not true. The SI units are based on standards established by an international agreement. For example, a meter is defined as the distance covered by light traveling at some very small fraction of a second.The other three statements are true:The SI system is universal (except for most people in the United States); it's used everywhere. The units of the SI system are easy to convert; you just have to change prefixes. The SI system is a decimal system; conversions involve moving the decimal point.

What does the level of the thermal activity depend on when the reactor fuel elements in which fission takes place and where the heat energy is released has the highest temperature in a nuclear power plant?

The average cooling fluid temperature According to second Law of thermodynamics level of activity is higher if the temperature of warmer fluid is higher. The interest is to minimize the temperature rise of cooling fluid.

Cameron observes that moss always seems to grow on one side of a tree but not on the other. What is the best hypothesis that can be formed from Cameron's observations?

The best hypothesis would be that the moss has some preference for one side of a tree over another. This is the simplest hypothesis. The assumption that the moss can get nutrients from only one side of a tree, or that the tree somehow determines where or how the moss grows is too complicated a hypothesis to establish.

What would be the most credible scientific evidence for establishing that global warming is occurring ?

The best scientific evidence is firsthand data showing that what you are claiming is true. A rise in atmospheric CO2 does not show that, but a rise in global temperature does.

Which of the following structures is not part of the central nervous system?

The brain and spinal cord are part of the central nervous system. The cerebrum and cerebellum are part of the brain. Motor neurons extend from the spinal cord to the muscle and are part of the peripheral nervous system.

Stellar parallax shift can be used to measure the distance between a star and the Earth. So when the parallax shift is larger, which of the following is occurring?

The closer the star is to Earth. The greater a star shift on a background of stars, the larger the parallax angle; and the closer that the star is to Earth. Through the application of trigonometry we are able to determine that the larger the shift angle, the shorter the distance to Earth.

In addition to the solar radiation that warms the surface of Venus, there are also thick clouds in the planet's atmosphere. Which of the following best explains how the clouds increase the surface temperature?

The cloud cover prevents the escape of heat into space The correct answer is (A). Venus' cloud cover increases the temperature of the planet surface because it does not allow heat to escape into space.

connect together so that the open centers of the protein rings, called pores, are aligned. form the walls of a little tunnel that passes between the two cells, and ions and small molecules can

The connected rings of proteins travel through the tunnels from one cell to the next. In response to signals, the protein rings can pull together, closing the opening to the tunnel and blocking the movement of materials.

In which of the traditional five taxonomic kingdoms is the jump from unicellularity to multicellularity believed to have occurred?

The consensus is that multicellular organisms evolved from colonies of unicellular protists. Many single-celled protists transferred genetic material through pores in their outer membranes. That function can be extrapolated to more highly organized multicellular protists that divided certain tasks to certain cell types. At some point, individual cells no longer needed to have locomotion and digestive capabilities; and for example, certain cells within a multicellular protist could be devoted to certain tasks. The correct answer therefore, is (B).

Plants, algae, and cyanobacteria all utilize carbon dioxide from Earth's atmosphere in which of the following processes?

The correct answer is (A). Photosynthesis, the conversion of light energy into chemical energy, is the process in which plants, algae, and cyanobacteria all utilize carbon dioxide from Earth's atmosphere.

Which of the following characteristics is measured by a Richter scale?

The correct answer is (A). The Richter magnitude scale assigns a single number to quantify the amount of seismic energy released by an earthquake by recording waves from the center of the disturbance.

Which of the following best describes the function of the rough endoplasmic reticulum?

The correct answer is (A). The rough endoplasmic reticulum is the membranous network that receives and transports synthesized proteins.

A meridian is a line of which of the following?

The correct answer is (B). A meridian line runs from north to south and refers to distance from the equator.

The three main components to the circulatory system are

The correct answer is (B). Although the lungs play an important role in the circulatory process because they oxygenate the blood, the three main components of the circulatory system are the heart, the blood and the blood vessels.

Which of the following statements is FALSE about asexual reproduction?

The correct answer is (B). Asexual reproduction is not limited to single-cell organisms. In fact, most plants have the ability to reproduce asexually.

A calcium deficiency would affect which part of the body the most?

The correct answer is (B). Calcium is an extremely important component in maintaining a strong skeletal system.

Cirrus clouds are usually

The correct answer is (B). Cirrus clouds are usually thin and wisp-like, almost to the point of forming a veil-like covering called cirrostratus.

Cumulonimbus clouds are most often associated with what kind of weather?

The correct answer is (B). Cumulonimbus clouds are frequently associated with heavy precipitation and thunderstorms.

Which of the following organs is responsible for the final stage of digestion, absorbing water and ridding the body of solid waste?

The correct answer is (B). The final stage of digestion occurs in the large intestine.

Which of the following agricultural processes improves soil structure and fertility by alternating deep-rooted and shallow-rooted plants?

The correct answer is (C). Crop rotation is the practice of growing a series of dissimilar types of crops in the same space in sequential seasons for various benefits such as to avoid the build up of pathogens and pests that often occurs when one species is continuously cropped.

Which of the following terms is most often used to describe trees that shed their leaves seasonally?

The correct answer is (C). Deciduous is a term that literally means "tends to fall off," and is most used when describing trees that shed their leaves during autumn, such as Maple and Elm trees.

What is the difference between metamorphosis and evolution? Metamorphosis is the change in a single animal in a single lifetime and evolution is the change in a species over many lifetimes

The correct answer is (C). Evolution, on a grand scale, is the change in a species over numerous lifetimes and metamorphosis, on a much smaller scale, is the change in a single animal in a single lifetime.

Which of the following organs is part of the excretory system?

The correct answer is (C). The sweat glands are part of the excretory system. The heart, brain and stomach are parts of the circulatory, nervous and digestive systems, respectively.

Ultraviolet radiation comes from light composed of

The correct answer is (C). Ultraviolet light is composed of only waves having a very short wavelength, shorter than blue or violet.

When classifying organisms through binomial nomenclature the seven main ranks from SMALLEST to LARGEST are as follows:

The correct answer is (D). From smallest to largest, the binomial nomenclature is Species, Genus, Family, Order, Class, Phylum, Kingdom

Which of the following is the correct order of the path of food after being swallowed?

The correct answer is (D). The correct order of the path of food directly after being swallowed is esophagus, stomach, small intestine, large intestine.

Which of the following is a type of magma that is located in the Hawaiian Islands?

The correct answer is basaltic. The types of magma are basaltic, andesitic, and rhyolitic.

A system is a group of interacting parts that form a whole. Which of the Earth systems identify how humans and their natural processes changes the Earth's atmosphere?

The correct answer is biosphere. The biosphere is one of the four main Earth systems.

Which of the following is a classification of sedimentary rock?

The correct answer is clastic. Sedimentary rocks are classified as clastic, chemical or biochemical.

Which of the following identifies a type of wind erosion?

The correct answer is deflation. Wind, as a powerful contributor to erosion, picks up and carries sediment. Further, wind has two types of erosion agents which are deflation and abrasion.

A gust of wind blows soil across an open field. This type of action is called

The correct answer is erosion. Erosion caused by wind is visible as the gust of wind blows soil across an open field. And erosion occurs when the force from the wind as it lifts sediment up from the ground and out of the rocks and then carries it away to other locations.

Which of the following equalizes the Earth's thermal energy?

The correct answer is global winds. Global winds are created by the coriolis effect and the Earth's heat imbalance.

Which of the following statements comparing granite intrusion to schist in the Grand Canyon in regards to a cross cutting relationship is correct?

The correct answer is granite intrusion in the Grand Canyon is younger than the schist. In the Grand Canyon, rocks that are in the deepest part of this canyon are made of igneous and metamorphic rock. Within the schist section of these igneous and metamorphic rocks are intrusions of granite called dikes. These dikes are located at the bottom or the rock and are considered to be the youngest layer of the rock. Also, since the granite intrusion cuts across the schist in the rocks, the rocks are identified as having a cross cutting relationship. Cross cutting relationships mean that an intrusion is younger than the rock that it cuts across.

Which of the following best identifies a type of mudflow?

The correct answer is lahar. Lahar is a type of mudflow. Typically, lahar occurs when a snow topped mountain volcano erupts and then melts the snow that is on top of the mountain. The melted snow and ash mixes together and are transported down slope.

Which of the following is an example of nonfoliated metamorphic rocks?

The correct answer is marble. Marble and quartzite are two common types of nonfoliated rocks.

Based on the image above of a fossil shell, which of the following develops when sediment covers the hard section of an organism?

The correct answer is mold. Fossil remains give scientists information about historical events of the Earth.

Which of the following statements represents the reason scientists use C-14 to calculate the age of organic material?

The correct answer is organic materials contain carbon. Scientists use the radioactive decayed isotopes, C-14 or Carbon-14, for calculating the age of organic materials because it contains a high concentration of carbon. The types of organic materials that scientist use for radiocarbon dating are amber, plant and animal based materials, and bones and charcoal.

The lunar surface has a system with elongated streaks that are brightly colored. These streaks are known as

The correct answer is rays. The blasted material from an impact with an object that falls back to the moon is classified as ejecta. The long trails of ejecta that is generated from these moon craters are known as rays, which are brightly colored streaks.

Which of the following magma types contains the highest silica content?

The correct answer is rhyolitic. Rhyolitic is a type of magma classification. Additional types of magma are basaltic and andesitic. The magma that has the highest silica content is rhyolitic at more than 66%.

Which of the following tides happen when the sun, the moon and the Earth align?

The correct answer is spring tides. The spring tides happen as the sun, the moon and the Earth are aligned. These tides have a great range in the winter months for regions in the northern hemisphere.

The ozone layer in the Earth's atmosphere plays a vital role in

The correct answer is the absorption of harmful radiation. The ozone layer blocks harmful radiation from reaching the Earth. Regulating the energy amounts absorbed by the atmosphere is incorrect as this describes the roles of variable atmospheric gases. Then, serving as a transitional space between outer space and Earth's atmosphere is a characteristic of the Earth's exosphere. Next, the absorption of solar radiation is a component of the Earth's thermosphere.

When the density and atmospheric pressure decreases, what happens to the altitude?

The correct answer is the altitude increases. When the density and the pressure of the atmosphere decreases, the altitude increases. This happens because the atoms and molecules move farther apart in the atmosphere.

Which of the following best infers why the sunspots on the sun are darker than its surrounding areas?

The correct answer is the darker areas are cooler. A sunspot is a dark feature that is located on the surface of the sun's photosphere. The sunspots are darker in some areas because of the temperature, which is 1500 K less than its surrounding areas. This temperature on the sun makes the sunspots cooler.

Glaciers form in places like Greenland and Antarctica. They also form high in mountains like the Alps and the Andes. Which of the following statements is NOT a necessary condition for glaciers to form?

The correct answer is: The air temperature can never get above the melting point of water for the entire year.

The planets can be divided into two major groups depending on how they are composed. The four planets closest to the sun are called:

The correct answer is: terrestrial planets, because they're made mostly of rock, like the Earth. Terrestrial means Earth-like. They are not called mercurial of martian. The four outer planets (not including Pluto) are Jovian - large gas balls like Jupiter.

Tornadoes can be measured according to their intensity. The scale was developed that is used today called the:

The correct answer is: "F"- scale, or Fujita scale. It's named after Dr. Theodore Fujita, who developed it. The Richter scale is for earthquakes; the Doppler scale is for radar, and the Messier scale really doesn't exist - he cataloged objects in space.

A child runs for 90.0 seconds at a speed of 3.0 meters per second. The distance the child travels is:

The correct answer is: 270 meters. Distance equals rate times time. So, 3.0 m/s times 90.0 seconds is 270 meters. The other answers are incorrect.

Constellations, like Orion shown above, are typically made up of different kinds of stars. In Orion, Rigel is a blue-white star, while Betelgeuse is a red giant. What can be said about the composition of most stars?

The correct answer is: Stars are nearly identical in composition to other stars - almost all are 73% hydrogen. Stars, therefore, do not vary widely in composition. The color differences primarily are due to the temperatures of each star, not their composition. Cooler stars can have titanium oxide in their atmospheres, but they are still about 73% H - 25% He. They are about 25% helium - not 90%.

The picture above shows an earthworm. To describe how an earthworm reproduces, you would say:

The correct answer is: earthworms are hermaphroditic, meaning every one is both male and female, so any earthworm can mate with any other. There are no separate male and female earthworms. They can regenerate lost halves, but that is typically not how they reproduce.

The human brain is made up of billions of cells. These cells that make up all nervous tissue in the human body are called:

The correct answer is: neurons. Neurons are nerve cells. Neutrons are neutral particles in the nucleus of an atom. Neoplasms are new growths caused by cancer cells. Nephridial cells are associated with the kidneys.

The Moon has many features on its surface that can be seen from the Earth. These features were caused by lava flows and by the impacts of meteors. Large, flat areas, called maria, are scarred by meandering, valley-like structures that might be collapsed lava tubes that are called:

The correct answer is: rilles. Rays are light-colored streaks of ejecta (material blasted out of impact craters) that are generally straight lines. Eskers are meandering ridges produced by melting glaciers. Aquifers are underground water flows under the Earth's surface.

Newton's Third Law of Motion states that for every action:

The correct answer is: there is an equal and opposite reaction. This is the law of action - reaction. Force equals mass times acceleration is the second law. And ojects in motion and objects at rest is the first law.

There used to be nine planets until Pluto was downgraded to a dwarf planet. Which of the following lists includes the eight remaining planets in their order from the sun?

The correct order of the eight remaining planets is Mercury, Venus, Earth, Mars, Jupiter, Saturn, Uranus and Neptune.

The Calvin cycle uses ATP and NADH to convert carbon dioxide to sugar. In the Calvin cycle the starting materials are regenerated. Carbon enters the cycle in the form of carbon dioxide and leaves in the form of sugar.

The cycle uses ATP for energy and consumes NADPH as reducing power. The light reactions regenerate this ATP and NADPH.

Which of the following compounds would contain the same number of particles as one mole of nitrogen? I. Carbon. II. Helium. III. Sulfuric acid.

The definition of one mole is the amount of substance which has the same amount of elemental components or particles

Activation energy is the energy required to bring the reactants up to the level that is needed to proceed. What is the difference in energy between the activated complex and the products?

The difference between the energy of the reactants and the peak of the activated complex is the

While scuba diving, Andrew noticed that the fish he speared was one-third (1/3) smaller after he brought it to the surface than when he saw it underwater. What is the reason for that? The depth of the water makes the fish bigger. Fish always seem bigger when you first catch them. Light waves travel faster in water than in air. The diffraction of light in water make the fish appear bigger.

The diffraction of light in water make the fish appear bigger. Explanation: The slower speed of light in water causes a diffractive difference between the air in Andrew's mask and the water that makes the fish appear one-third (1/3) larger. The diffraction of light between air and water makes everything underwater seem about one-third (1/3) larger.

Peristalsis is what? Correct answer: Muscle contractions that move food through the digestive tract

The digestive tract of a mammal consists of the alimentary canal (digestive tract) and accessory glands that aid with digestion. Rhythmic muscle contractions push food through the digestive tract. Sphincters regulate the passage of material between segments of the digestive tract.

A body moving in a straight line with an initial velocity of 5 m/s and a constant acceleration covers a distance of 30 m by the third second. How much distance will it cover in the next two (2) seconds?

The distance covered in the nth second is given as sn= u+a(n-1/2). Therefore, 30= 5+(3-1/2)a, which gives a=10m/s2. s4= 5+ 10 x (4-1/2)= 40m and s5= 5+ 10 x (5-1/2)= 50m. Thus, the distance traveled in the 4th and 5th seconds is 40+50=90m.

A cube of side b has a charge q at each of its vertices. What is the electric potential at the center of the cube? Correct answer: 4q/(√3πε0b).

The distance of a vertex from the center of the cube of side b is r=√3b/2. Now the potential due to charge q at the center is q/(4πε0r). Hence, the potential due to the arrangement of eight charges at the center V=8q/(4πε0r)= 4q/(√3πε0b).

Imagine a movie theater full of people. When the movie is over, everyone leaves but they must all exit through 2 doors in the back of the theater. few people every second, so there is a maximum rate at which the people can exit through the doors.

The doors can only let through a Which of the following cell processes is comparable to this illustration? Correct answer: facilitated diffusion

Of the 20 basic amino acids, how many cannot be synthesized by the human body, and are therefore considered to be essential?

The eight (8) essential amino acids are: Isoleucine, leucine, lysine, methionine, phenylalanine, threonine, tryptophan, and valine. Eight (8) others can be considered as being "essential" under certain circumstances. Amino acids are the building blocks of all proteins that perform countless functions in the biological system. The correct answer therefore, is (C).

This is a free electron in a uniform field E(vector). The electron moves in what direction and what will happen with the electric potential energy?

The electron will move to the left and the electric potential energy will decrease. Electrons experience a force in opposite direction of the field and their electric potential energy decrease during this movement.

How much energy is required to heat 1 kg of water from 20oC to 40oC? For the specific heat of the water, you may use 1.0 cal/g oC.

The energy required is (1000 g) (1.0 cal/g oC) (40 - 20 oC) = 20,000 calories. A joule is a unit of work or energy. The units in the question are given in calories, and provide a clue toward the solution to the question.

The experiment calls for the addition of gram negative stain to a slide to stain some bacteria. What happens if the addition of the stain does not take place?

The experiment may not be considered valid. Not following lab procedure carefully can result in invalidation of test results.

A horizontal overhead power line carries a current of 90A in an east to west direction. What is the magnitude of the magnetic field due to the current of 1.5 m being below the line?

The expression for the magnitude of the magnetic field is μ0I/2πr= 4π•10-7•90/2•π•1.5=1.2x10-5 T.

A wire coil is held with its plane horizontal to the Earth's surface and a small bar magnet is dropped vertically down through it. How will the magnet fall?

The falling magnet induces a current in the coil. From Lenz's Law, the direction of the current is such that its magnetic field opposes the motion of the magnet. It falls therefore, with a non-uniform acceleration less than g.

Which statement is not a possible definition for an acid? definition of an acid. The second is the Lewis definition of an acid.

The first definition is the Arrhenius The third is the Lewis definition of a base (thus it is NOT a definition for an acid, and is the answer to the question ). The fourth is the Bronsted-Lowery definition of an acid.

Which of the following statements are true for a chemical compound?

The first statement is true for an element, but not for a compound. The second statement is true for both an element and for a compound. The third statement is sometimes used as the definition of a compound. The fourth statement is true for a mixture, but not for a compound.

A student is hoping to produce a quantity of sodium chloride, and according to her research she expects a theoretical yield of 135g from the reaction chosen.

The first time she performs the reaction she manages a yield of 59% of what was expected. After making some adjustments, she manages to improve the yield amount by 24g. What is the yield amount that she manages in the second experiment?

For most of history, it was believed that the sun was actually a burning ball of fire in the sky. We now know that the power of the sun comes from which of the following?

The fusion of hydrogen and other atoms to make heavier elements is what powers the sun. Hydrogen atoms are fused to form helium, and helium atoms fuse to form other heavier elements. Differences in the type of star will determine what other types of elements are being produced, but hydrogen and helium must always be present, or the star will not be "burning".

Which of the following is NOT a function of the Golgi apparatus? Correct answer: Destruction

The golgi apparatus is a center for manufacturing, warehousing, sorting and shipping in the cell. Products from the endoplasmic reticulum arrive at the golgi apparatus where they are modified and stored, then sent to other locations.

The biosphere includes the organisms that are on the Earth. Also, the biosphere includes the environments that organisms live.

The humans in the biosphere change the atmosphere through their activities and through natural processes.

Trip found the definition of an isotope in his science book, and it said that an isotope is an element that does which of the following?

The isoptopes of an element are atoms that have the same number of protons, and different numbers of neutrons. Hydrogen for instance, has one proton and deuterium. An isotope of hydrogen has one proton with one added neutron.

In science class you observe a red liquid. According to optical theory, the reason that the liquid is red is attributable to which of the following?

The liquid reflects only red light and absorbs all other colors of light. That is why you see it as red.

When excess fluid cannot be collected by capillaries in the circulatory system, which of the following organ systems is primarily responsible for returning the fluid to the circulatory system?

The lymphatic system is similar to the circulatory system in that it contains many vessels that extend to various areas of the body. One of its functions is to remove excess fluid from peripheral tissue. It collects excess fluid, and returns it to the circulatory system through the thoracic duct in the chest. The correct answer therefore, is (D).

When would the magnifying power of a compound microscope be high?

The magnifying power of a compound microscope is given by M=(L/fo)(1+D/fe), in which L is the length of the microscope. For M to be high, both fo and fe must be small.

Uranium has 92 protons and 146 neutrons. The total protons and neutrons represents what number?

The mass number is the total of the protons and the neutrons. In the case of uranium, its mass number is 238 (92 protons and 146 neutrons).

Which of the following statements are true regarding enthalpy and entropy?

The meanings of the terms enthalpy and entropy are quite different, so A is false. The entropy of a perfect crystal at absolute zero is equal to zero, according to the Third Law of Thermodynamics, so entropy at other conditions can be determined from that reference point, and B is true. Only changes in enthalpy due to a process, or enthalpy of a substance relative to some defined reference condition can be determined, so C is false. Statement D is false. Changes in entropy and enthalpy for a process may have the same sign, or they may have opposite signs.

Sexual reproduction of water molds takes place in specialized structures that are formed by the hyphae. One strucutre, the antheridium, produces male nuclei.

The other structure, the oogonium, produces female nuclei. Fertilization, or sexual fusion, occurs within the oogonium, and the spores that form develop into new organisms.

The yield of a chemical reaction is the actual amount of a compound which is produced.

The percentage yield of a reaction is calculated by dividing the yield by the theoretical maximum yield and then multiplying the answer by 100.

When do the properties of real gas become quite similar to the properties of ideal gas? High temperature and low pressure

The predictions of 'ideal gas' model deviates more from observation at low temperature andmodels are very similar.

Which of the following properties in metals does not result from the presence of metallic bonding? Correct answer: Magnetism

The presence of metallic bonding accounts for strength, malleability, thermal and electrical conductivity, ductility, luster and opacity. In contrast, magnetism arises as a result of the orbital angular motion of the electrons in certain metals.

Which of the following protein features does the tertiary structure of a protein refer to? Correct answer: the overall 3-D shape

The primary structure is the sequence of amino acids, and the secondary structure is the local folding and coiling. The function can vary from protein to protein, so it is not given a generic name. The tertiary structure is the overall 3-D shape.

What is the resistivity of mica? Correct answer: 1 x 1013 ohm/m

The resistivity of mica (insulator) is 1 x 1013 ohm/m whereas, the other choices are resistivity of glass, copper and Quartz, respectively.

The first process forms the new substances, carbon dioxide and water, from the organic matter in the wood, and the fourth process forms a new substance, sodium chloride. So those two are chemical processes.

The second process merely converts carbon dioxide from solid to gaseous form, and the third process converts water from liquid to gaseous form. No new substance is formed in either of those, so they are physical changes.

Which of the following statements are true for a saturated solution of calcium fluoride?

The solubility is affected by temperature. Usually, the solubility is higher at increased temperatures. So decreasing the temperature will cause calcium fluoride to excrete from a solution. The other three statements are true.

Which type of magma is found along oceanic-continental subduction zones?

The source material for andesitic magma is oceanic crust or oceanic sediments. Thus, this magma is found along the oceanic-continental subduction zones.

How does the sun produce energy?

The sun like every star undergoes fusion reactions which involve hydrogen combining to form helium under immense temperatures and pressure. Energy is released here as the difference in mass of the hydrogen atoms and helium atom is converted to energy.

What is the difference between smooth endoplasmic reticulum (smooth ER) and rough endoplasmic reticulum (rough ER)?

The surface of smooth ER lacks ribosomes The cytoplasmic surface of smooth ER lacks ribosomes. Rough ER appears rough under a microscope due to the fact that it has ribosomes on its cytoplasmic surface.

How can internal resistance be identified?

The value of EMF of a battery will always be lesser than the potential difference Whenever internal resistance id present in a battery, it is identified by the value of battery's potential difference, as the potential difference is fund to be lesser than the battery's EMF.

Which of the following describes what a vapor point is?

The vapor point is the same as the boiling point of a substance. It is different for each substance, depending on pressure and temperature.

Because males only have one x chromosome, they have to inherit the chromosome from their maternal side.

The x chromosome does not usually express itself in a female unless they get two bad x chromosomes, which is extremely rare.

Foliated metamorphic rock and nonfoliated rock are incorrect answer choices as these are classifications of metamorphic rocks.

Then, quartzite is an incorrect answer choice as this is a type of nonfliated metamorphic rock that is hard and lightly colored.

Which of the following best describes a population's carrying capacity?

There are enough resources in a population's habitat to support all individuals. For example, microorganisms will continue to reproduce until they reach the carrying capacity of a given environment. The correct answer choice therefore, is (A).

Which statement is true concerning an acid-base titration?

There are several possible indicators to use for acid-base titrations, with different pHs at which their color change takes place, so the first statement is true.

Which statement is true concerning an acid-base titration?

There are several possible indicators to use for acid-base titrations, with different pHs at which their color change takes place, so the first statement is true. The indicator should be chosen so that the end point is approximately the same as the equivalence point (at which equivalents of acid = equivalents of base), but the indicator used has no effect on the equivalence point. If a weak acid and/or weak base is used, then the equivalence point pH will not be 7.0. It would take less of a higher concentration titrant to reach the end point, not more.

Which of the following hypotheses of DNA replication is correct? It holds that each strand of DNA gives rise to and is paired with one completely new strand of DNA.

There were competing hypotheses regarding the formation of DNA including the conservative, semiconservative and dispersive hypotheses. In the conservative hypothesis, both

The equilibrium constant expression for this reaction is [Products] / [Reactants].

Therefore, (500)2 / (.05) (.03). This will be a very large equilibrium constant at those concentrations. To reach equilibrium of 2.5 X 103 , the reaction must shift to the left.

Which is the correct ionic formula for a molecule of sulfite? Most oxygen-containing anions end in the suffix, '-ate', but if there is a similar molecule with less oxygen ions, the correct suffix to use is '-ite'.

Therefore, SO42- is sulfate while SO32- has one less oxygen atom, is sulfite. Neither of those is associated with a single negative charge; but rather are both associated with a charge of 2-.

A type of texture that is defined by well formed crystals that are surrounded with finer grained crystals is known as The correct answer is porphyritic texture. Porphyritic texture is the texture of a rock that has large, well developed crystals.

These developed crystals are surrounded by finer grained crystals that have the same or a different type of mineral. Vesicular texture is an incorrect choice as this a rock that has a spongy appearance.

Above are fossilized cyanobacteria. Most scientists think that these organisms could have produced enough oxygen to change the composition of the atmosphere.

These large, coral reef-like mounds of cyanobacteria are called: The correct answer is: stromatolites.

What is anabolism? Chemical process that synthesize body tissues Anabolism refers to chemical processes which are constructive, or synthesizing of body tissues, enzymes and other body components.

These processes coexist, however one may dominate at times over the other. For example, at night during sleep, anabolic processes tend to dominate, while catabolism tends to dominate during the day.

Stanley Miller and Harold Urey demonstrated that amino acids could be synthesized under conditions that approximate the early Earth namely water, methane, ammonia, and hydrogen in the presence of an electrical spark (lightning).

These scientists reported five amino acids but later work demonstrated that 22 were possible. This experiment demonstrated the principle of chemical evolution.

What is meant by the following: chloroplasts and mitochondria are semiautonomous organelles?

They grow and reproduce within the cell Mitochondria and chloroplasts grow and reproduce within the cell. They also contain a small amount of DNA.

What is meant by the following: chloroplasts and mitochondria are semiautonomous organelles? They grow and reproduce within the cell. They can survive outside the cell. They can carry out their functions without the need of the other functions or products of the cell. Their membranes are non permeable, thus they do not share products with the cell.

They grow and reproduce within the cell. Mitochondria and chloroplasts grow and reproduce within the cell. They also contain a small amount of DNA.

What is the molar concentration of a sodium hydroxide solution made up by dissolving 800 mg of NaOH in water and diluting it to 500 ml? (You may use 40.0 as the molecular weight of NaOH.)

This can be calculated as: (0.8 g)/(40 g/mole) = 0.02 moles and (0.02 moles)/(0.5 liters) = 0.04 moles/liter = 0.04 M solution.

Consider an object moving between points A and B. When does the magnitude of its velocity become greater than its speed? When the body moves in a straight line between A and B This can never happen. When the body takes a curved path and the angle of the tangent at any point along the path and the radius is 36° When the distance between points A and B is the unit vector

This can never happen. The magnitude of velocity of a body is the displacement divided by the time taken. The displacement between points A and B is a constant. If the body travels in a straight line between A and B its speed is equal to the magnitude of its velocity. For any other path the speed is greater than the magnitude of its velocity. The magnitude of its velocity can therefore never be greater than its speed.

All of the following are ways to produce an embryo EXCEPT: harvesting The traditional method of embryo production involves the union of egg and sperm cells at fertilization to produce a zygote.

This zygote becomes an embryo as a result of cleavage. Cloning and parthogenesis are two biotechnological methods of embryo production.

Using his lab data, Brian calculates sea water density to be 0.9 g/cm3. He has been told that the density of distilled water is about 1.0 g/cm3. What should his next step be?

To double-check his results for reasonableness Good lab procedure always dictates checking the reasonablenness of results. Because of its salt content, sea water has to be more dense than distilled water, but the results show the opposite. Something is in error.

Kim looked out the window of her school room and noticed big, black clouds in the distance heading toward her. What would she expect the barometer in the school to do as the storm approaches?

To drop rapidly The storm would bring a dropping barometer with it. The faster it drops, the worse the storm. This is because storms are created when warm air meets cold air. The warm air is forced upwards, and collides with the cool air, resulting in wind and forming storm clouds. When warm air is forced upwards, it leaves a low-pressure area surrounded by high-pressure air. Low surface pressure makes barometers drop.

In DNA replication, which are example of proteins responsible for unwinding the DNA?

Topoisomerase and helicase The topoisomerase protein is critical to relaxing the tension of the tightly coiled DNA molecule so that it can be straightened out and separated by DNA helicase. Without this protein there would be too much tension on the DNA molecule. Imagine trying to straighten a coiled telephone cord (from a corded telephone!). You can push it just so far until there is too much stress on the cord. Topoisomerase makes tiny cuts in the DNA backbone to release this strain without fully unraveling the DNA. Polymerase is and enzyme associated with replication and transmission. Ligase is an enzyme that is a catalyst in forming a chemical bond. Annealase is an activity which alters.

A ray of light travels from a medium of refractive index, n1 to a medium of refractive index, n2. When would the total internal reflection be possible?

Total internal reflection is an optical phenomenon that occurs when a ray of light strikes a medium boundary at an angle larger than the critical angle with respect to the normal to the surface. If the refractive index is lower on the other side of the boundary, (i.e. n1>n2), light cannot pass through, and all of the light is reflected. The critical angle is the angle of incidence above which the total internal reflection occurs.

Phages can transfer genes from one bacterium to another by a process called what? Transformation. Translocation. Transduction. Transposons.

Transduction. Explanation: Correct answer: Transduction Phages can transfer genes from one bacterium to another by a process called transduction. In generalized transduction, a random piece of host DNA is accidentally packaged within a phage capsid and then incorporated into a new host bacterium.

Name the periods within the Mesozoic Era in correct chronological order.

Triassic, Jurassic, and Cretaceous is the correct order of periods within the Mesozoic Era. Each period lasted about 60 million years, and to this day remain very well known for dinosaurs.

In this diagram, if our results fall within the black shaded area, and we rejected the null hypothesis, it would be called what? Type II error. Type I error. Correct decision. Statistical error.

Type I error A type I error is also called a false positive. It occurs when a statistical test rejects a true null hypothesis (H0).

If the potential difference between the plates of a capacitor is increased by 20 percent (20%), what is the increase in energy stored in the capacitor?

U1=½CV2. U2=½C(1.2V)2= ½•CV2•1.44. Therefore (U2-U1)/U1= (1.44-1)/1= .44= 44%. The energy stored in the capacitor increases by 44%.

Which waves manifest weak diffraction?

Ultrasound waves Ultrasound waves have the high intensity and small wavelength so they manifest the weak diffraction. We can get moderate beams of ultrasound waves.

How can spherical aberration in a thin lens be reduced?

Using a circular, annular mask over the lens To reduce spherical aberration, the aperture must be decreased with an annular mask.Using monochromatic light eliminates chromatic aberration. Using a double combination minimizes chromatic aberration. Increasing the size of the lens increases its resolving power.

Two objects of mass m1 and m2 are travelling with velocities u1 and u2 respectively and after colliding they travel with the velocity v1 and v2. Which equations are correct in calculating v1 and v2?

V1 = [ u1 {(m1 - m2)/ (m1 + m2)} + 2 m2u2 /(m1 + m2)] The algebra required to find this solution is left to the reader. The physics involved in this sort of collision are nicely explained in Wikipedia: http://en.wikipedia.org/wiki/Momentum

A gazelle finds itself being chased by a cheetah. All of the following are reactions of the circulatory system EXCEPT:

Vasodilation in the circulatory system refers to the opening of blood vessels to allow greater blood flow, while vasoconstriction is the narrowing of blood vessels to restrict blood flow. The respiratory and muscular systems both use more blood during the fight-or-flight response, and blood flow to the epidermis is restricted. However, the digestive system is not necessary right away, so the small intestine does not need increased blood flow.

The physical property that describes a material's resistance to flow is called

Vicosity describes a material's resistance to flow. Temperatue and silica content affect the viscosity of a magma. Cooler magma resists flow whereas warmer magma flows easily.

Which term is not a unit of heat?

Watt is a unit of power. Heat which has the same units as work can be expressed in calorie, Joule and BTU or British Thermal Unit. One BTU is approximately 1054 to 1060J and one calorie is 4.2J.

A phase change diagram illustrates which of the following?

What phase a substance is in at a specific temperature and pressure. A phase diagram illustrates what phase a substance is in at a specific temperature and pressure. That is valuable information because each substance is different.

Identify the term that defines anything that is not matter. Our understanding of the world is that everything is either matter (that which takes up space and has mass) or energy.

When a nuclear reaction takes place, matter is converted to energy. Anti-matter, while the opposite of matter, can still be defined as a form of matter since it has mass and takes up space.

When a solute is moved across the cell membrane against, or "up" a concentration gradient, this is known as what? Correct answer: Active transport

When a solute is moved across the cell membrane against, or "up" a concentration gradient, this is known as active transport. This type of transport requires work and the cell must expend its own energy to accomplish it.

What does the Archimedes' principle state? According to Archimedes' principle, "Any object, wholly or partly immersed in a fluid, is buoyed up by a force equal to the weight of the fluid displaced by the object."

When an object is immersed in water it is pushed up by a force equal to the weight of the water displaced by the object. Therefore, an object will be lighter immersed in water than outside it.

Parts and function of the brain: Cerebrum - memory, creativity, intelligence Cerebellum - balance and coordinating controllable actions Medulla - reflexes for the upper part of the body and control over heart and respiratory system Spinal Cord - lower body reflexes

Which part of the brain helps a student win a spelling bee? cerebellum cerebrum spinal chord medulla Explanation: The correct answer is (B). The cerebrum controls memory and intelligence, which are necessary to spell words correctly.

Which of the following is not considered a storage molecule?

While starch, glycogen and triacylglycerol are sufficient for the storage of "fuel" in cells, pyruvate is a rapidly used. When pyruvate is formed in the cell it is destined to be made into something else rather quickly. Pyruvate should not be thought of as a storage molecule.

During the Cambrian Period, what is North America today was an ancient called Laurentia. There was no tectonic activity along the edges everywhere on Laurentia. This kind of continental edge is referred to as:

a passive margin. It wouldn't be active, because an active margin is where there IS some tectonic activity. A geothermal fissure is involved with tectonic activity - magma moving out from underneath a tectonic plate caused by its movement.

The Golgi apparatus, or the Golgi body, is defined as a structure that assists with cell division the membranous network in the endoplasmic reticulum the membranous network that synthesizes lipids and carbohydrates a stack of flattened, membranous sacs that chemically processes and packages substances from the endoplasmic reticulum

a stack of flattened, membranous sacs that chemically processes and packages substances from the endoplasmic reticulu The Golgi apparatus is a stack of flattened, membranous sacs that chemically processes and packages substances from the endoplasmic reticulum

All of the following statements about the plasma membrane are true EXCEPT:

about 1% of the plasma membrane is made up of proteins The plasma membrane is made up of two layers of phospholipids along with proteins, sterols, and carbohydrates. Phospholipids make up about 50% of the plasma membrane, carbohydrates are attached to receptors on the outside of the plasma membrane, and sterols are also embedded in the membrane. The type of sterol depends on the type of cell. Proteins are stuck in the membrane and associated with the edge of the membrane. Proteins make up almost 50% of the plasma membrane

A large amount of air that has the characteristics of the area from which it develops is called a(n)

air mass.

thus disputing the idea of spontaneous generation. Hooke studied nonliving cells. Leeuwenhoek was the first to see bacteria from teeth scrapings and

animals-like protists from pond water, and Schleiden worked with plants and noticed that all plants are made of cells.

During sexual reproduction in water molds, which of the following structures produces male nuclei?

antheridium

high pressure in which the attraction between molecules becomes significant and the size of the molecules can no longer be

approximated to zero when compared to volume of gas. But at high temperature and low pressure, predictions of both

All of the following statements about radioactive isotopes are true EXCEPT:

are not dangerous Radioactive isotopes have unstable nuclei and break down at a constant rate over time. The radiation give off by these isotopes can be dangerous, but they can also be used to determine the ages of rocks and fossils and also used as tracers to follow the movements of substances with organisms.

Valence electrons

are what give atoms their properties because of how they are transferred or shared

Based on the image above, which of the following is the most flexible renewable resource? bamboo copper wind steel

bamboo. Bamboo grows fast and does not need to be replanted. Due to these characteristics, bamboo is classified as a resource with sustainable yield.

A type of sand dune that creates a solitary crescent shape is which of the following?

barchan. A barchan dune creates solitary crescent shapes. Further, the barchan dunes are developed from small amounts of sand and are located in areas that have small amounts of vegetation or no vegetation.

The study of volcanoes is called Volcanism. Volcanism describes all processes associated with the discharge of three of the following; which one is NOT true?

basalt. Basalt is rock that was formerly molten; it has cooled slowly; it is not discharged from a volcano. The other answers are true - Volcanism is about the discharge of magma, hot fluids, and gases.

In organic chemistry nomenclature, what substance of a carboxyl group (-COOH) is added to a hydrocarbon chain? Correct answer: Acid. Described here, carboxylic acids are the acids of organic chemistry,

because they are able to lose the hydrogen from the COOH group. Alcohols are formed when a hydroxyl group is found on a hydrocarbon. Ketones include a carbonyl group. Organic bases are always weak bases, and can be formed from ammonia groups.

Trees in the tropical rain forest are tall with broad evergreen leaves. Four different layers exist based on their need for light. the top layer is the canopy and requires the most light;

below is the subcanopy and then the understory. The forest floor is the lowest layer and receives the least amount of light.

Where does most of the Earth's volcanism occur?

below the Earth's surface Not all magma reaches the Earth's surface. Due to the interaction between the rising magma and the Earth's crust several things can happen. The overlying rock can be forced apart, blocks of rock can break off and sink into the magma, eventually melting the rock. Magma can also melt its way through the rock.

The cross product of two vectors, a and b, is equal to c. What is always true of c? c has a magnitude always greater than a+b c is a vector. c points in a direction which bisects the angle between a and b. c is a unit vector.

c is a vector. Explanation: When the cross product of two vectors a and b is determined the result is a vector given by ||a|| ||b|| sin(θ) n , where n is a unit vector perpendicular to both a and b which completes a right-handed system. Therefore c=a x b is always a vector. The other options may be true or false depending on the values of the vectors a and b.

The tropical rain forest biome contains which of the following biotic factors?

canopy

The remarkable growth rate of mushrooms is caused by which of the following?

cell enlargement Fruiting bodies expand with astonishing speed, sometimes producing fully developed mushrooms overnight. This growth rate is caused by cell enlargement, not cell division. The cells of the hyphae enlarge by rapidly taking in water.

The "protein factory" of the cell is the ribosome, which is made of RNA. The mitochondrion is the "powerhouse" of the

cell where the majority of the cell's adenosine triphosphate (ATP).

Which of the following forces doubles the thickness of the deformed crust in continental-continental convergence?

compressional forces Explanation: The correct answer is compressional forces. Compressional forces breaks the earth's crust into thick pieces. These pieces are then thrown at each other along low angle faults. As a result of this process, the thickness of the deformed crust is doubled.

Which of the following are concerned with the protection and maintenance of ecosystems?

conservationist A conservationist is concerned with the protection and maintenance of ecosystems and their biological communities.

Based on his experiments Galileo concluded that all mechanical experiments done inside a ship moving at a

constant speed in a constant direction would give precisely the same results as similar experiments done on shore.

This type of magma has the same composition as granite - more than 60% silica. It occurs when molten material rises and mixes with the

continental crust above it which would be rich in silica and water. Because of its high viscosity, eruptions can be explosive.

Scientists believe the main force for plate movement is which of the following?

convection Convection currents are one of the main theories of the Earth's mantle that causes plates to move.

Scientists believe the main force for plate movement is which of the following? ridge push slab pull hot spot convection

convection Convection currents are one of the main theories of the Earth's mantle that causes plates to move. Convection is the motion of matter including thermal energy that results from the changes in temperature. Ridge push is an incorrect choice as this is a result of the weight of the lifted ridge pushing the oceanic plate to the trench that is created at the subduction zone. Further, slab pull is an incorrect choice as this is the weight of the subducting plates that pulls the slab trailing it into the subduction zone. Then, hot spot is an incorrect choice as a hot spot is the heat in the mantle that has the ability to produce magma, which rises to Earth's surface.

The picture above, from the Kachina Bridge sandstone at the Natural Bridges National Park in Utah, shows layers of rock that must have been laid down by moderate to fast-moving water. There are strong current marks where individual layers are caught up in cross-currents. This picture shows an example of:

cross-bedding. Cross-bedding occurs when rock layers form at different angles. "Environments in which water movement is fast enough and deep enough to develop large scale bed forms" is how cross-bedding is defined in Geologywiki. Graded bedding is caused by different sizes of grains being deposited in layers with the largest grains on the bottom and the smallest grains on the top. Volcanic flow wouldn't form sedimentary layers - these layers are made of sandstone, not any igneous rocks. Superposition is the theory that earlier rocks are deposited below later rocks. Oldest rocks are at the bottom of a rock column... youngest rocks are at the top.

Which of the following did the Earth's molten rock form?

crust The formation of the early Earth included a period of cooling on the Earth's surface. From this cool down, the minerals and elements in magma concentrated as a dense zone known as the crust.

A child is riding a merry go round and her horse is at the top of its leap. In which direction is the child accelerating?

downward and toward the center of the merry go round The child is accelerating downward at the top of the horse's leap. At all times, she is accelerating toward the center of the merry go round.

The above picture shows a landscape of a mountain. If a continental glacier moved over older moraines of this mountain and form material into elongated landforms, which of the following best describes this process?

drumlins. Drumlins are shaped when glaciers move over old moraines.

Precipitation refers to the forms of condensation that fall to earth. Which of the following is NOT a form of precipitation? dust rain sleet snow

dust Explanation: The correct answer is (A), dust because it does not fit the definition of precipitation.

All of the following groups have an exoskeleton EXCEPT:

echinoderms An exoskeleton is a shell and insects and mollusks have shells. Arthropod is a phylum that includes both insects, arachnids, and crustaceans, so this group also has an exoskeleton. Echinoderms are organisms like starfish and sea urchins, which have an endoskeleton.

The process where sediment is removed and transported is known as erosion bedding grading crossing

erosion Explanation: The correct answer is erosion. Erosion is a process where weathered sediment is carried from one location to another location. Sediment is typically transported through water, glaciers, wind and gravity. Erosion typically happens after the rock fragments are removed from the rocks through the weathering process.

Which of the following algae phyla are plantlike protists that have two flagella but no cell wall?

euglenophytes Members of the phylum Euglenophyta are closely related to the animal-like flagellates. They are plantlike protists that have two flagella but no cell wall. Although euglenophytes have chloroplasts, in more other ways they are like zooflagellates. Euglenas are found in ponds and lakes throughout the world and they are excellent swimmers. If sunlight is not available, euglenas can also live as heterotrophs, absorbing the nutrients available in decayed organic material

Of the following scenarios, which is most likely to result in directional selection? females prefer males with the best tail symmetry females prefer males with the loudest mating call females prefer males that are solid grey females prefer older males

females prefer males with the loudest mating call Directional selection means the most extreme version of a trait results in reproductive success. The trait needs to be the biggest, brightest, smartest, or some other -est, in the case of this question, the loudest.

Which three cell parts are most likely to be found in sperm cells? flagellum, mitochondria, nucleus flagellum, chloroplast, nucleus mitochondria, ribosomes, nucleus cilia, mitochondria, nucleus

flagellum, mitochondria, nucleus Explanation: Correct answer: flagellum, mitochondria, nucleus Sperm carry genetic information (DNA) in a nucleus, and swim using a flagellum. Swimming takes a lot of energy, so mitochondria break down fuel and produce ATP.

Both schist and gneiss(as seen above) are coarse-grained, foliated metamorphic rocks. Both have minerals arranged in parallel bands, but gneiss is different from schist because:

gneiss has well-developed banding due to alternating layers of different minerals, usually of different colors. Gneiss has well-developed banding due to alternating layers of different minerals, usually of different colors. Schist usually appears as dark because it mostly comes from shale. Both schist and gneiss have banding patterns, but the patterns in gneiss are more pronounced. They don't differ from each other much in hardness, and neither one is typically shiny.

One way to form rock layers is for water that is supersaturated with minerals to be trapped in a place where the water can evaporate. These are called evapoite deposits. Two kinds on rock layers that are evaporite deposits are:

gypsum and halite. Gypsum is an evaporite made of calcium sulfate dihydrate; halite is an evaporite made of salt (sodium chloride). Shale is not an evaporite - it is formed from clay deposits, which don't dissolve in water. Sandstone is also not an evaporite, since it is formed from sand, which doesn't dissolve in water, either. Granite is an igneous rock; gneiss is metamorphic. Basalt and pumice are also igneous rocks.

According to the above diagram, the number 7 identifies which of the following on the microscope? The disc diaphragm on a microscope typically

has 5 holes of different diameters.Each one changes the amount of lighting passing through, and it is used to properly illuminate specimens and improve resolution and contrast.

Which of the following may happen to the body if sodium and potassium ion levels are abnormal?

heart attack If the level of sodium and potassium ions is abnormal in the body, the ability for muscles to contract or for nerves to send impulses correctly is affected, which can interfere with the heartbeat and potentially cause a heart attack.

Which of the following develops as trade winds create high pressure areas? intertropical convergence zone horse latitudes tropical zones prevailing westerlies

horse latitudes Horse latitudes are created from the sinking air near latitudes 30 degrees north and 30 degrees south where the trade winds form high areas of pressure. Additionally, horse latitudes have a belt of weak surface winds. Deserts, such as the Sahara desert, is located under this high pressure horse latitude area

Which of the following develops as trade winds create high pressure areas?

horse latitudes.

All of the following factors are involved in determining biotic potential EXCEPT: the age of organisms when they're able to reproduce. how many offspring are male. how often the organisms reproduce. the number of offspring that survive to adulthood.

how many offspring are male. The maximum growth rate of a population under ideal conditions is referred to as biotic potential. Ideal conditions occur when species don't have to compete for resources, such as food or water, and when no predators or diseases affect the growth of the organisms. The sex of an organism's offspring is not necessary to determine the biotic potential.

Stars are made up mostly of which of the following? helium hydrogen plasma water

hydrogen Explanation: The correct answer is: hydrogen Stars are made by a fusion of hydrogen and helium, about 75% of which is hydrogen.

Global warming can be attributed to which of the following? development of more algae in the oceans increased consumption of fossil fuels development of new plant life decreasing emissions of carbon dioxide

increased consumption of fossil fuels Explanation: The correct answer is (B). A main cause of global warming is humanity's increased consumption of fossil fuels.

A type of technique that produces an image from several telescopes is called cartographometry geometry interferometry astrometry

interferometry Explanation: The correct answer is interferometry. A process called interferometry helps improve the resolution of images that have been converted from data by a computer. Interferometry uses the images from many telescopes to generate one image.

Scientists use which of the following principles to study the order that geological events happen? relative age dating The correct answer is relative age dating. Relative age dating is a principle that

is used to determine the relative age, for instance of a fossil, in order to determine when and how geological events happened. However, relative age dating does not give scientist an exact number of years that an event occurred but more of an insight about geological events that have happened.

Which of the following is an invertebrate, or does not have a backbone? tiger jellyfish eagle alligator

jellyfish Explanation: The correct answer is (B). The jellyfish is the only invertebrate. All the others are vertebrates.

Of the following organs, which is specifcally a part of the excretory system? heart salivary glands cerebrum kidney

kidney Explanation: The correct answer is (D). The kidney is the organ in which the body wastes collect. These wastes then form urine and move on to the urinary bladder.

Which of the following is the daughter product to the radioactive parent isotope of thorium?

lead. Lead is a daughter product of the radioactive parent isotope called thorium. Scientists use the decay of radioactive isotopes in rocks to determine the age of a rock.

This type of locomotion is the most energy efficient. Swimming Overcoming gravity is less of a problem with swimming due to the buoyancy of water,

making swimming a highly efficient means of locomotion. Water is dense and creates a great deal of resistance, therefore fast swimmers have a body shape that reduces friction.

The four phyla of animal-like protists are distinguished from one another by which of the following?

means of movement The four phyla of animal-like protists are distinguished from one another by their means of movement. Zooflagellates swim with flagella, sarcodines move by extensions of their cytoplasm, ciliates move by means of cilia, and sporozoans do not move on their own at all.

A person's RNA can be found in all of the following locations within a cell EXCEPT:

mitochondria Genetic RNA can be found in a cell's nucleus, cytoplasm, and rough endoplasmic reticulum, but RNA is not found in mitochondria. Mitochondria have their own DNA and RNA which are separate from nuclear genetic material

A person's RNA can be found in all of the following locations within a cell EXCEPT: nucleus cytoplasm mitochondria rough endoplasmic reticulum

mitochondria Genetic RNA can be found in a cell's nucleus, cytoplasm, and rough endoplasmic reticulum, but RNA is not found in mitochondria. Mitochondria have their own DNA and RNA which are separate from nuclear genetic material

The complexity of the behavior of an animal is directly related to the complexity of the animal's ___________ system. nervous circulatory skeletal excretory

nervous Explanation: The correct answer is (A). Behavior in animals is controlled by the endocrine system and the nervous system.

Which of the following is a type of chromosome mutation?

nondisjunction mutation Chromosome mutations occur when the DNA changes. Nondisjunction is the failure of a pair of chromosomes to separate during meiosis, and the result is a change in chromosome number. One gamete has an extra chromosome and the other gamete is missing a chromosome.

How is the nuclear envelope able to allow molecules to move to and from the rest of the cell?

nuclear pores The nuclear envelope is dotted with thousands of nuclear pores, which allow material to move into and out of the nucleus.

The air temperature can and does get above the melting point of water at times during the year. As long as the snow doesn't melt completely over the summertime,

or the total thickness doesn't decrease from year to year, glaciers can form. A glacier is produced when the weight of the top layers of snow produce enough pressure to melt and recrystalize the lower layers into ice.

When fossils do not have their original material from the original organism, then these fossils are classified as mold. Mold develops when sediments cover the

original hard surface of an organism, such as a hard shell. Then the hard portion is removed through erosion or weathering.

Which of the following best represents the principle where the layers of sediment are generally deposited in a horizontal layer?

original horizontality. This principle states that the layers of sediment are deposited in a horizontal position. Gravity, along with wind and water, are responsible for spreading the sediment in this manner.

In which of the following ways do bryophytes transport water? diffusion osmosis translocation vascular tissue

osmosis Bryophytes have only one way to transport water and that is from cell to cell by osmosis. This fact limits their height to just a few centimeters; for millions of years, plants grew no larger.

Which of the following did scientists use to determine if oxygen was present in the Archean atmosphere?

oxidized iron The correct answer is oxidized iron. Scientists looked for oxidized iron in Archean rocks to determine if there was oxygen in the Earth's atmosphere during the Archean time. Since there were no iron oxides in the rocks in the Archean period, scientists determine that the Archean time did not have oxygen in its atmosphere.

Which of the following processes is the conversion of light energy into chemical energy? photosynthesis decomposition respiration distillation

photosynthesis Explanation: The correct answer is (A). Photosynthesis is the process through which plant life converts light energy into chemical energy using chlorophyll.

Which of the following statements is true of both Darwinian evolution and Lamarckian evolution? fitness depends on the number of viable offspring produced competition prevents evolution from occuring populations change over time to be better adapted to their environments an individual may acquire an adaptive trait during its own lifetime and pass that trait on to its offspring

populations change over time to be better adapted to their environments This is the only statement that both Darwin and Lamarck agree on.

In eukaryotes, what are the final products of transcription and translation? primary transcript (pre mRNA) and protein. Protein and primary transcript (pre mRNA). transfer RNA (tRNA) and protein. Protein and rRNA.

primary transcript (pre mRNA) and protein The resulting product of transcription is pre mRNA. In eukaryotes, the mRNA must exit the nucleus before translation can occur. Thus, RNA processing, or the modification of pre mRNA occurs within the nucleus prior to the mRNA's exit from the nucleus. The final outcome, or product of translation, is protein. Ribosomes facilitate the pairing of tRNA anticodons with mRNA codons during protein synthesis.

Which of the following organisms were predominant on the Earth up until the end of the Precambrian?

prokaryotes. The organisms that were predominant on Earth in the Precambrian time were unicellular organisms. These organisms were prokaryotes, which means they did not have a nucleus.

Which of the following organisms were predominant on the Earth up until the end of the Precambrian? cyanobacteria ediacarans fungi prokaryotes

prokaryotes. The organisms that were predominant on Earth in the Precambrian time were unicellular organisms. These organisms were prokaryotes, which means they did not have a nucleus.

In human beings, oxygen is delivered to tissues throughout the body via __________ in the blood. white blood cells red blood cells plasma carbon dioxide

red blood cells Explanation: The correct answer is (B). Red blood cells, which are the most common type of blood cells, are the body's principle means of carrying oxygen throughout the body.

A type of telescope that has lenses to collect light is known as? array telescope refracting telescope reflecting telescope orbiting telescope

refracting telescopes. There are two types of telescopes that scientists use to focus viewable light: the refracting telescope and the reflecting telescope. The refracting telescope uses lenses to focus the visible light.

Which of the following is one of the main sources of carbon dioxide in the ocean? respiration photosynthesis decomposition erosion

respiration Erosion and decomposition do not release carbon dioxide into the ocean, and photosynthesis releases oxygen. Respiration from animals in the ocean are the main source of carbon dioxide in the ocean.

The blood pH of a small mammal has dropped. Activity in which of the animal's systems can correct the situation? nervous system digestive system respiratory system circulatory system

respiratory system The small mammal has excess CO2 in its blood, which made the pH level in its blood drop. In order to get rid of extra CO2, the small mammal needs to increase its breathing rate, which is exhaled through the respiratory system. Once the extra CO2 has been exhaled, the blood pH will go back up.

Which of the following structures is used to anchor black bread mold to the bread?

rhizoids Black bread mold has two different kinds of hyphae. The rootlike hyphae that penetrate the bread's surface are rhizoids. Rhizoids anchor the fungus to the bread, release digestive enzymes, and absorb digested organic material. The stemlike hyphae that run along the surface of bread are stolons. The hyphae that push up into the air are the sporangiophores, which form sporangia at their tips.

Which of the following is an example of water pollution? nitrogen oxides toxic dumps landfillls sewage

sewage Sources of water pollution include wastes from factories, farms, suburban areas, sewage and air pollutants. Landfills and toxic dumps can pollute land (soil), and nitrogen oxides are types of air pollutants that come from the burning of fossil fuels in cars and factories.

Magma is comprised of many compounds. Which of the following compounds in magma has the greatest effect on the magma's characteristics? silica aluminum potassium magnesium

silica Explanation: The correct answer is silica. Silica is the most abundant compound found on magma. Also, silica has the greatest effect on the characteristics of magma. Additional compounds found in magma are oxygen, aluminum, iron, magnesium, calcium, potassium, and sodium.

What will be the hybrid state of C in carbon disulphide?

sp S = C = S (linear) is best explained by sp-hybrid state of Carbon.

A scientist who studies caves is known as speleologist earth scientist environmental scientist meteorologist

speleologist Explanation: A speleologist is a scientist who studies caves. These scientist also use scientific methods to collect samples and observe the landforms that occur through a geologic process.

The product of a stream's average width, depth and velocity determines stream bed load stream depth stream discharge stream velocity

stream discharge Explanation: The correct answer is stream discharge. Stream discharge is the measure of the volume of stream water that flows past a particular location within a given amount of time. It is expressed in cubic meters per second.

(i.e. atoms or molecules) as there would be in 12g of carbon-12 (which is 6.02 x 10 ). That means that the mole mass of each of those

substances would be significantly different, but the number of component atoms or molecules would and must be the same in each.

Areas of reduced temperature on the sun are called sunspots hotspots solar flares starspots

sunspots Explanation: The correct answer is (A). A sunspot is a region on the sun's surface that is marked by a lower temperature than its surrounding regions and is the location of intense magnetic activity, which is the source of solar flares, among other phenomena.

Three experiments have been performed in a parallel fashion. Each of the reactions has taken different times to be completed in various steps. If it becomes necessary to show graphical representation of the time (t) difference versus change in

temperature (ΔT), which one will go along the X-axis? Correct Answer: Δt Temperature changes with the change of time, that is why Δt will go along X-axis.

A scientist starts out with 100g of hydrochloric acid and 20g of magnesium hydroxide. It was expected that the reaction would produce the salt magnesium chloride, and

that the maximum theoretical mass produced would be 18g. The salt which the reaction actually produced was weighed and found to be 16g. What is the percentage yield of the reaction?

The Moon's surface always appears to be identical to humans observing it on Earth. The fact that the same side of the Moon always faces the Earth can be explained by the fact that:

the Moon has a synchronous rotation - it rotates with a period equal to its orbital period. The Earth's gravitational pull doesn't affect the Moon's rotation - and the Moon is rotating at the same rate it revolves around the Earth. The Sun also does not stop the Moon from rotating. The Moon does have its own gravitational pull - and it is rotating. Some scientists theorize that Earth's gravity slowed the Moon's original spin until it reached synchronous rotation.

A major factor that causes limestone to dissolve is the acidity of precipitation the chemical reaction of oxygen with iron human activities that generate erosion water molecules with silicate minerals

the acidity of precipitation Explanation: The correct answer is the acidity of precipitation. One major factor that causes limestone to dissolve is natural precipitation. Precipitation includes rain, snow, sleet and fog. Further, natural precipitation is a pH of 5.6 which makes it acidic. The slight acidity of precipitation is what dissolves the limestones and rocks similar to it.

activation energy for the forward reaction. The difference between the energy of the products and

the activated complex is the activation energy for the reverse reaction.

The equation for the formation of water plus the equation for the reaction in question is equivalent to the equation for the formation of hydrogen peroxide. Thus, per Hess's Law, the sum of the enthalpies of formation of water and

the enthalpy of reaction for formation of H2O2 from H2O, and O2 must equal the heat of formation of hydrogen peroxide. Thus, the heat of reaction for the equation, H2O(l) + ½ O2(g) → H2O2(l), must be -187.8 - (-285.8) kcal = +98 kcal.

The enzyme binds to its substrate and while they are bound, the catalytic action of

the enzyme converts the substrate to the product of the reaction.

new strands annealed with each other after replication while the semiconservative model held that one new and one old strand of DNA are paired. The dispersive hypothesis suggested that

the new and old fragments were randomly dispersed throughout the new molecule. Meselsohn and Stahl provided the evidence that DNA replication takes place by the semiconservative route.

The continuity equation is a mathematical statement that, in any steady state process,

the rate at which mass enters a system is equal to the rate at which mass leaves the system.

A chemical reaction occurs inside of a beaker and the sides of the beaker become hot. Which of the following statements is true about the chemical reaction? the reaction is very fast the reaction is exothermic the reaction is very slow the reaction is helped by an enzyme

the reaction is exothermic Thermic refers specifically to heat energy and exo means outside, so exothermic reactions release energy into their surroundings and in this case would make the beaker feel warmer. The speed of the reaction was not mentioned in the question, so the speed of the reaction is unknown. It is also unknown whether the reaction was helped by an enzyme simply by feeling the sides of the beaker.

During the early formation of the Earth, what broke apart the concentration of ammonia and methane in the Earth's atmosphere? the sun's ultraviolet radiation. Scientists speculate that during the early formation of the Earth,

the sun's ultraviolet radiation released more hydrogen into the atmosphere. This radiation was believed to have broken down the accumulating ammonia and methane gases that were in the atmosphere at that time.

The image above depicts sunlight. Which of the following statements best explains why the sun is classified as a renewable resource?

the sunlight is available for billions of years. The Earth is not the only source for renewable energy. The sun provides energy for every aspect of the Earth's processes. Also, since the sun's energy can be replenished in a short amount of time, it is considered renewable energy.

When an infected mosquito bites a human, the mosquito's saliva, which contains sporozoites, enters the human's bloodstream. Once inside the blood, Plasmodium infects liver cells and

then red blood cells, where it multiplies rapidly. When the red blood cells burst, the release of the parasites into the bloodstream produces severe chills and fever, symptoms of malaria.

Above is a model of an Aluminum atom. The orbits in a three-dimensional model are called shells. The outer shell electrons are called valence electrons. Valence electrons are important because

they are involved in chemical bonding, either being transferred or shared. Valence electrons are what give atoms their properties because of how they are transferred or shared.

All of the following statements about plant roots are true EXCEPT:

they store glucose The root of a plant anchors the plant to the ground and serves to support the stem. The root absorbs water and minerals and send these materials up through the stem to the leaves. The roots also serve as a major storage site of starch.

Which of the following junction types is important in surface tissues like skin? adherens junction anchoring junction tight junction surface junction

tight junction Tight junctions bring cells together so tightly that not even water can pass between the cells. Proteins pass through the membranes of both cells, holding the cells like they were sown together. The proteins are arranged in lines so that continuous bands of attachment are formed like seams of stitches through the tissues. Tight junction are important in surface tissues like skin because they create a great barrier to molecules and foreign organisms that would otherwise slip between the cells.

Broken-down food substances eventually reach all the cells of an elephant by way of the blood stream. Which of the following functions does this describe? transport digestion locomotion synthesis

transport Transport is the life function by which materials such as food, water, and oxygen from the environment are distributed to all cells of an organism. In animals, the blood circulatory system is responsible for carrying food substances to all parts of the organism.

From the Earth's surface, the atmosphere can be divided into four major divisions vertically. Going from lowest to highest, the layers are:

troposphere, stratosphere, mesosphere, and thermosphere. The lowest layer, where we live, is the troposphere. The next layer up is the stratosphere. Then the very large "middle layer", the mesosphere. At the top is the very hot thermosphere.

Which of the following structures of a compound microscope supports the eyepiece and the nosepiece?

tube The tube, or barrel, supports the eyepiece and the nosepiece. The ocular, or eyepiece is a convex lens used to magnify the image. The arm is used to carry the microscope and the stage supports the glass slide and contains the specimen being observed.

Which of the following is NOT a component of cellular respiration? Glycolytic decay

vCellular respiration is a cumulative function of three metabolic processes: 1) glycolysis 2) Krebs cycle 3) Electron transport chain. The energy released during each step of the chain is stored in a form the mitochondrion can use to make ATP for the cell.

Which of the following are used to bring DNA into other cells?

vectors A vector is a carrier used to bring DNA into other cells. Viruses and bacterial plasmids are examples of commonly used vectors.

Which of the following is a single-stranded RNA molecule that has no surrounding capsid? viroid prion bacteria virus

viroid Many plants, including potatoes, tomatoes, apples, and citrus fruits can be infected by viroids. Viroids are single-stranded RNA molecules that have no surrounding capsids. It is believed that viroids enter an infected cell and direct the synthesis of new viroids. The viroids then disrupt the metabolism of the plant cell and stunt the growth of the entire plant.

There are four forms of precipitation. Sleet forms by:

water droplets form in a warm (above freezing) cloud; they coalesce, then come down through air that is below freezing. Sleet is liquid water first, then freezes.

Each individual hydrogen bond is weak, but many hydrogen bonds together do important things. For example, hydrogen bonds between

water molecules give water its surface tension, which is just enough cling to support the weight of small water bugs.

In general, as the amount of dissolved gases in lava increase, the explosive nature of an eruption increases. The most common dissolved gas in the magma of a volcano is:

water vapor. The ground water in the rocks around the magma chamber gets into it seeps into the magma.

In the Freshwater biome, which of the following has the greatest amount of diversity?

wetlands Freshwater biomes include ponds, rivers, streams, lakes, and wetlands. Wetlands, in particular, have the greatest amount of diversity.

What is the potential energy stored in a spring that has a spring constant k and is compressed by a distance x?

½kx2 The potential energy stored in a spring is given by ½kx2. This comes from adding up the energy it takes to incrementally compress the spring.

The displacement of a body from a reference point is √x= 2t+3 in which (x) is in meters and (t) is in seconds. Identify the motion of the body.

√x= 2t+3. Squaring both sides, x=4t2+12t+9. As displacement x changes with time, t the body, is not at rest. Differentiating with respect to t, dx/dt=8t+12. The body therefore, is not in uniform motion as its velocity changes with time. Also, the velocity increases with time, and that shows that it is being accelerated.


Related study sets

Chapter 38 assessment and management of patients with rheumatic disorders prep u

View Set

Government and Civics: Semester 1 Exam

View Set

Fundamentals Nursing Prep U Chapter 14 Implementing

View Set

D adv Ch. 40: Endocrine Function

View Set

PE 150 Final Exam (Practice Tests)

View Set